Download as pdf or txt
Download as pdf or txt
You are on page 1of 52

The Brigham Intensive Review of

Internal Medicine Question & Answer


Companion 2nd Edition Ajay K. Singh
Visit to download the full and correct content document:
https://ebookmass.com/product/the-brigham-intensive-review-of-internal-medicine-qu
estion-answer-companion-2nd-edition-ajay-k-singh/
Any screen.
Any time.
Anywhere.
Activate the eBook version
of this title at no additional charge.

Expert Consult eBooks give you the power to browse and find content,
view enhanced images, share notes and highlights—both online and offline.

Unlock your eBook today.


1 Visit expertconsult.inkling.com/redeem Scan this QR code to redeem your
eBook through your mobile device:
2 Scratch off your code
3 Type code into “Enter Code” box

4 Click “Redeem”
5 Log in or Sign up
6 Go to “My Library”
Place Peel Off
It’s that easy! Sticker Here

For technical assistance:


email expertconsult.help@elsevier.com
call 1-800-401-9962 (inside the US)
call +1-314-447-8200 (outside the US)
Use of the current edition of the electronic version of this book (eBook) is subject to the terms of the nontransferable, limited license granted on
expertconsult.inkling.com. Access to the eBook is limited to the first individual who redeems the PIN, located on the inside cover of this book,
at expertconsult.inkling.com and may not be transferred to another party by resale, lending, or other means.
2015v1.0
Second Edition

The
Brigham
Intensive
Review of
Internal
Medicine
QUESTION & ANSWER COMPANION

Ajay K. Singh, MBBS, FRCP, MBA


Senior Associate Dean
Global and Continuing Education
Harvard Medical School
Physician, Renal Division
Brigham and Women’s Hospital
Boston, MA

Sarah P. Hammond, MD
Assistant Professor of Medicine
Harvard Medical School
Division of Infectious Diseases
Department of Medicine
Brigham and Women’s Hospital
Boston, MA

Joseph Loscalzo, MD, PhD


Hersey Professor of the Theory and Practice of Physic
Harvard Medical School
Chairman, Department of Medicine
Physician-in-Chief
Brigham and Women’s Hospital
Boston, MA
1600 John F. Kennedy Blvd.
Ste 1800
Philadelphia, PA 19103-2899

THE BRIGHAM INTENSIVE REVIEW OF INTERNAL MEDICINE


QUESTION & ANSWER COMPANION, SECOND EDITION ISBN: 978-0-323-48043-7

Copyright © 2019 by Elsevier, Inc. All rights reserved.

No part of this publication may be reproduced or transmitted in any form or by any means, electronic or me-
chanical, including photocopying, recording, or any information storage and retrieval system, without permis-
sion in writing from the publisher. Details on how to seek permission, further information about the Publisher’s
permissions policies and our arrangements with organizations such as the Copyright Clearance Center and the
Copyright Licensing Agency, can be found at our website: www.elsevier.com/permissions.

his book and the individual contributions contained in it are protected under copyright by the Publisher (other than
as may be noted herein).

Notices

Knowledge and best practice in this ield are constantly changing. As new research and experience broaden our
understanding, changes in research methods, professional practices, or medical treatment may become necessary.
Practitioners and researchers must always rely on their own experience and knowledge in evaluating and using
any information, methods, compounds, or experiments described herein. In using such information or methods
they should be mindful of their own safety and the safety of others, including parties for whom they have a profes-
sional responsibility.
With respect to any drug or pharmaceutical products identiied, readers are advised to check the most current
information provided (i) on procedures featured or (ii) by the manufacturer of each product to be administered,
to verify the recommended dose or formula, the method and duration of administration, and contraindications.
It is the responsibility of practitioners, relying on their own experience and knowledge of their patients, to make
diagnoses, to determine dosages and the best treatment for each individual patient, and to take all appropriate
safety precautions.
To the fullest extent of the law, neither the Publisher nor the authors, contributors, or editors, assume any liabil-
ity for any injury and/or damage to persons or property as a matter of products liability, negligence or otherwise,
or from any use or operation of any methods, products, instructions, or ideas contained in the material herein.

Previous edition copyrighted 2014 by Oxford University Press.

Library of Congress Cataloging-in-Publication Data

Names: Singh, Ajay, 1960- editor. | Loscalzo, Joseph, editor.


Title: he Brigham intensive review of internal medicine question & answer
companion / [edited by] Ajay K. Singh, Joseph Loscalzo.
Description: Second edition. | Philadelphia, PA : Elsevier, [2019] | Includes
index.
Identiiers: LCCN 2017042502 | ISBN 9780323480437 (pbk. : alk. paper)
Subjects: | MESH: Internal Medicine--methods | Physical Examination--methods
| Problems and Exercises
Classiication: LCC RC46 | NLM WB 18.2 | DDC 616--dc23 LC record available at
https://lccn.loc.gov/2017042502

Executive Content Strategist: Kate Dimock


Senior Content Development Specialist: Joan Ryan
Publishing Services Manager: Catherine Jackson
Book Production Specialist: Kristine Feeherty
Design Direction: Patrick Ferguson

Printed in China
Last digit is the print number: 9 8 7 6 5 4 3 2 1
Contributors

Amy Bessnow, MD Ole-Petter R. Hamnvik, MB BCh BAO, MMSc


Instructor in Medicine Assistant Professor of Medicine
Harvard Medical School Harvard Medical School
Department of Medicine Division of Endocrinology, Diabetes, and Hypertension
Brigham and Women’s Hospital Department of Medicine
Dana-Farber Cancer Institute Brigham and Women’s Hospital
Boston, MA Boston, MA
Hematology and Oncology Endocrinology

Robert Burakof, MD, MPH Galen V. Henderson, MD


Vice Chair for Ambulatory Services Assistant Professor of Medicine
Department of Medicine Harvard Medical School
Weill Cornell Medical College Department of Neurology
New York, NY; Brigham and Women’s Hospital
Site Chief Boston, MA
Division of Gastroenterology and Endoscopy General Internal Medicine
New York–Presbyterian Lower Manhattan Hospital
New York, NY; Jennifer A. Johnson, MD
Visiting Scientist Assistant Professor of Medicine
Harvard Medical School Harvard Medical School
Boston, MA Division of Infectious Diseases
Gastroenterology Department of Medicine
Brigham and Women’s Hospital
Elizabeth Gay, MD Boston, MA
Member of the Faculty of Medicine Infectious Diseases
Harvard Medical School
Division of Pulmonary and Critical Care Medicine Ann S. LaCasce, MD
Department of Medicine Associate Professor of Medicine
Brigham and Women’s Hospital Harvard Medical School
Boston, MA Department of Medical Oncology
Pulmonary and Critical Care Medicine Dana-Farber Cancer Institute
Department of Medicine
Sarah P. Hammond, MD Brigham and Women’s Hospital
Assistant Professor of Medicine Boston, MA
Harvard Medical School Hematology and Oncology
Division of Infectious Diseases
Department of Medicine Ernest I. Mandel, MD
Brigham and Women’s Hospital Instructor in Medicine
Boston, MA Harvard Medical School
Infectious Diseases Division of Renal Medicine
General Internal Medicine Department of Medicine
Brigham and Women’s Hospital
Boston, MA
Nephrology and Hypertension

iii
iv Contributors

Muthoka L. Mutinga, MD Scott L. Schissel, MD


Assistant Professor of Medicine Instructor in Medicine
Harvard Medical School Harvard Medical School
Division of Gastroenterology, Hepatology, and Endoscopy Chief, Department of Medicine
Department of Medicine Brigham and Women’s Faulkner Hospital
Brigham and Women’s Hospital Division of Pulmonary and Critical Care Medicine
Boston, MA Brigham and Women’s Hospital
Gastroenterology Boston, MA
Pulmonary and Critical Care Medicine
Anju Nohria, MD, MSc
Assistant Professor of Medicine Lori Wiviott Tishler, MD
Harvard Medical School Assistant Professor of Medicine
Division of Cardiovascular Medicine Harvard Medical School
Department of Medicine Division of General Internal Medicine and Primary Care
Brigham and Women’s Hospital Department of Medicine
Boston, MA Brigham and Women’s Hospital
Cardiovascular Disease Boston, MA
General Internal Medicine
Molly Perencevich, MD
Instructor in Medicine Derrick J. Todd, MD, PhD
Harvard Medical School Instructor of Medicine
Division of Gastroenterology, Hepatology, and Endoscopy Harvard Medical School
Department of Medicine Division of Rheumatology, Immunology, and Allergy
Brigham and Women’s Hospital Department of Medicine
Boston, MA Brigham and Women’s Hospital
Gastroenterology Boston, MA
Rheumatology
Megan Prochaska, MD
Research Fellow in Medicine
Harvard Medical School
Department of Medicine
Brigham and Women’s Hospital
Boston, MA
Nephrology and Hypertension
Preface

Preparing for the American Board of Internal Medicine their eforts and commitment to this project. We asked
(ABIM) certifying or recertifying examination requires them to put themselves “in the head” of the ABIM to
knowledge and clinical experience that can be evaluated identify the topics that might be addressed in the board
by successfully answering questions in a test format. In examination. We believe this book will be a valuable
this Question and Answer book, our goal is to provide the study tool to gauge one’s knowledge in preparation for the
reader with 450 questions across 9 subspecialties in inter- examination.
nal medicine. hese questions test knowledge on topics rel- We wish to thank Stephanie Tran and Michelle Deraney
evant to the ABIM boards. As a companion to the Brigham for supporting us in the development of this book. Without
Intensive Review of Internal Medicine, now in its third edi- them this book would not have been possible. Our thanks
tion, this book is focused on how one applies knowledge to also go to our families who have supported all of our aca-
answer board questions successfully. he annotated answers demic activities, including this important project.
are detailed, and they review the steps in critical thinking
required to get to the correct answer. Ajay K. Singh, MBBS, FRCP, MBA
he authors who have contributed questions and anno- Joseph Loscalzo, MD, PhD
tated answers to this book are some of our most senior Sarah P. Hammond, MD
physicians in the department. We sincerely thank them for

v
Contents

1. Infectious Diseases, 1
Sarah P. Hammond and Jennifer A. Johnson

2. Hematology and Oncology, 28


Amy Bessnow and Ann S. LaCasce

3. Rheumatology, 49
Derrick J. Todd

4. Pulmonary and Critical Care Medicine, 78


Scott L. Schissel and Elizabeth Gay

5. Endocrinology, 104
Ole-Petter R. Hamnvik

6. Nephrology and Hypertension, 129


Megan Prochaska and Ernest I. Mandel

7. Gastroenterology, 145
Muthoka L. Mutinga, Molly Perencevich,
and Robert Burakof

8. Cardiovascular Disease, 165


Anju Nohria

9. General Internal Medicine, 191


Lori Wiviott Tishler, Sarah P. Hammond,
and Galen V. Henderson

Index, 212

vi
1
Infectious Diseases
SARAH P. HAMMOND AND JENNIFER A. JOHNSON

1. A 28-year-old woman who has lived her entire life in donor for polycystic kidney disease and has had no
Providence, Rhode Island, presents 3 days after return- episodes of rejection since. Her donor was cytomega-
ing from a 2-week trip to hailand complaining of lovirus (CMV) immunoglobulin G (IgG) negative,
fever to 102°F, muscle aches, and severe retroorbital and she was CMV IgG positive before transplant. Her
headache. She has no gastrointestinal symptoms. She chronic medications for transplant include tacrolimus,
traveled only to the towns of Bangkok, Chiang Mai, low-dose prednisone, and mycophenolate mofetil and
and Phuket. She attended a travel clinic before travel- have not changed in several years. She is a third-grade
ing and was told there was no malaria in these towns, teacher and has recently been taking care of the class
so she did not take prophylaxis. She denied contact pets, which include two goldish and a hamster. On
with bodies of fresh water. Examination is unremark- presentation she has a fever to 102.1°F and has difuse
able other than temperature of 101.8°F. Remarkable tenderness of the abdomen without rebound.
laboratory indings include a leukocyte count of 2200 he most likely cause of her present illness is:
cells/µL3, hematocrit of 37%, and platelets of 62,000 A. Salmonellosis
cells/µL3. Chemistries are normal. A peripheral blood B. Medication-induced diarrhea related to the cumu-
smear for parasites is sent and is negative. lative efects of mycophenolate
Which of the following is the most likely diagnosis C. Cytomegalovirus colitis
in this traveler? D. Norovirus gastroenteritis
A. Leptospirosis E. Irritable bowel syndrome
B. Malaria
C. Typhoid 4. A 24-year-old man with ulcerative colitis presents in
D. Hepatitis A January for a irst primary care clinic visit with you as
E. Dengue his new primary care physician. He was diagnosed with
ulcerative colitis involving the entire colon 10 months
2. A 55-year-old male smoker with severe chronic ago and initially was treated with corticosteroids and
obstructive pulmonary disease (COPD) is hospitalized mesalamine. In the last 3 months he has been doing
in the medical intensive care unit. He now requires well on mesalamine and azathioprine, which are his
intubation and mechanical ventilation for hypercarbic only medications. He works as a paralegal and hopes
respiratory failure after failing noninvasive ventilation. to attend law school in the next few years. He lives
To reduce this patient’s risk for developing ventila- with his girlfriend of 2 years with whom he is monoga-
tor-associated pneumonia, you recommend: mous. He uses condoms for birth control. He is feeling
A. Elevation of the head of the bed to 15 degrees to well today. Physical examination is unremarkable; he is
prevent aspiration afebrile and well appearing. You review his immuniza-
B. Suctioning of subglottic secretions tion history—he has not received any vaccines within
C. Twenty-four hours of prophylactic systemic anti- the past 6 years; his last vaccination was the conjugate
biotics, especially if the intubation was emergent meningococcal vaccine at age 18.
D. Daily changing of the ventilatory circuit In addition to vaccinating for inluenza and human
E. Nasotracheal intubation rather than orotracheal papillomavirus, he should also receive which of the
intubation following vaccines?
A. Tetanus, diphtheria (Td) vaccine
3. A 47-year-old woman with a history of renal transplan- B. Pneumococcal 13-valent conjugate (PCV13) vaccine
tation presents with 4 days of profuse nonbloody diar- C. Haemophilus inluenzae B vaccine
rhea, abdominal pain, and high fevers. She received her D. Pneumococcal 23-valent polysaccharide vaccine
kidney transplant 10 years ago from a living unrelated E. Meningococcus B vaccine

1
2 C HA P T E R 1 Infectious Diseases

5. A 36-year-old man is found to have a positive tuber- hepatitis B DNA not detected, hepatitis C IgG nega-
culosis interferon gamma release assay result as part tive. She was treated with six cycles of rituximab,
of a workplace screening program. He is originally cyclophosphamide, doxorubicin, vincristine, and
from Bangladesh and was vaccinated with the bacil- prednisone (R-CHOP), which ended several weeks
lus calmette-Guérin (BCG) vaccine during childhood. ago and achieved complete remission based on posi-
He immigrated to the United States 6 months ago. He tron emission tomography (PET) CT imaging. Basic
reports feeling well. He has no fever, cough, or weight laboratory indings when she presents now are notable
loss. Physical examination is normal. for aspartate aminotransferase (AST) 527 U/L, alanine
he next best step in his management should be: aminotransferase (ALT) 495 U/L, total bilirubin 3.5
A. Sputum for smear microscopy and mycobacterial mg/dL, with a normal international normalized ratio
culture (INR). Her past history is notable for immigrating to
B. Initiation of isoniazid prophylaxis to prevent reac- the United States from rural Vietnam 2 years ago. She
tivation of latent tuberculosis infection had a PPD skin test at the time she immigrated and
C. Chest x-ray to assess for active pulmonary disease she was treated for latent tuberculosis with a 9-month
D. Perform a tuberculin skin test (puriied protein course of isoniazid that inished before the diagnosis of
derivative [PPD] test) to conirm the skin test lymphoma.
result A likely cause of her abnormal liver function tests
and malaise is:
6. A previously well 62-year-old man presents to the hos- A. Hepatitis C infection resulting from blood trans-
pital with increasing weakness in his lower extremities. fusion
Examination reveals decreased relexes symmetrically, B. Recurrence of her lymphoma
which progresses proximally over the course of several C. Acute hepatitis A infection
hours. He is diagnosed with Guillain–Barré syndrome D. Delayed isoniazid toxicity
and admitted to the intensive care unit for treatment. E. Reactivation of hepatitis B infection
On history, he reports several days of nausea, vomit-
ing, and diarrhea approximately 2 months prior. 9. A 34-year-old teacher presents to her primary care
he most likely infectious cause of his gastrointes- physician with 1 week of severe cough. Her symptoms
tinal illness was: began 2 weeks prior with a mild fever and rhinorrhea.
A. Campylobacter She has been experiencing posttussive emesis every
B. Giardia couple of hours. She takes levothyroxine for hypo-
C. Salmonella thyroidism but otherwise has no chronic illnesses.
D. Cryptosporidium Whooping cough is suspected, and a polymerase chain
E. Escherichia coli O157:H7 reaction (PCR) of a respiratory specimen is sent to test
for Bordetella pertussis.
7. A 24-year-old woman calls your oice complaining of he best management is:
burning with urination, and increased urinary urgency A. Treat with codeine-containing cough syrup for
and frequency. She reports no fever, nausea, vomiting, symptom control and wait for conirmation of B.
or lank pain. She has been in a monogamous rela- pertussis infection.
tionship for 3 years, and she had one prior episode of B. It is too late to treat for B. pertussis with antibiot-
cystitis, more than a year ago. ics; reassure her that the cough will improve in the
Which of the following agents is the best treatment next 2–4 weeks and administer the TDaP vaccine
for acute uncomplicated cystitis? now.
A. Cephalexin 500 mg twice daily for 7 days C. Start empiric azithromycin for a 5-day course.
B. Ciproloxacin 250 mg twice daily for 3 days D. Start empiric levoloxacin for a 7-day course.
C. Nitrofurantoin macrocrystals 100 mg twice daily E. It is too late to treat for B. pertussis with antibiot-
for 5 days ics; administer immunoglobulin to provide passive
D. Amoxicillin 500 mg three times daily for 7 days immunity.

8. A 65-year-old woman with recently diagnosed difuse 10. A 57-year-old man from lower Delaware presents to
large B-cell lymphoma presents with malaise, nausea, an urgent care center after being bitten by a tick. He
and mild jaundice. She was diagnosed with difuse reports that he spent many hours in his garden over the
large B-cell lymphoma after developing massive right weekend and was bitten by many insects. He returned
cervical lymphadenopathy and daily fevers 7 months to his work as an accountant after the weekend and
ago. She was profoundly anemic when she presented has been mostly spending time inside since then. his
and required a blood transfusion at that time. Pretreat- morning, a Tuesday, he noticed an engorged tick at
ment work up revealed the following: HIV antibody/ his waist line, which he removed. At the moment he
antigen negative, hepatitis A IgG positive, hepatitis B feels well other than worrying about getting sick from
surface antigen negative, hepatitis B core IgG positive, this tick. He has had no fevers, rashes, joint pains, or
CHAPTER 1 Infectious Diseases 3

headache. He brought the removed tick with him, and team notes that this is the third patient with C. diicile
it appears to be an engorged deer tick. infection on their unit in the past 2 weeks.
he best management plan is: To help reduce further horizontal transmission, you
A. Ceftriaxone 250 mg intramuscular × 1 recommend:
B. Amoxicillin 500 mg by mouth three times a day A. Identiication and treatment of asymptomatic car-
for 14 days riers
C. Check Lyme disease serologies, and treat with B. Strict adherence to standard precautions, includ-
antibiotics if the serology is positive. ing hand hygiene with an alcohol hand rub
D. Doxycycline 100 mg by mouth twice a day for 14 C. Use of a chlorine-containing cleaning agent to
days address environmental contamination
E. Doxycycline 200 mg by mouth × 1 D. Contact precautions for any patient suspected of
having C. diicile infection until 48 hours of ther-
11. A 53-year-old woman with well-controlled non– apy have been given
insulin-dependent diabetes and obesity develops an E. Prophylactic metronidazole for all patients on the
area of swelling and pain on the right thigh. Over unit
about 3 days, the area becomes luctuant and she
develops a large area of erythema around it. She 14. A 67-year-old woman with end-stage renal disease is
presents to the emergency room, where she is found admitted from hemodialysis with hypoxia, fever, hypo-
to have a low-grade fever but is otherwise stable. tension, and cough. She is diagnosed with multifocal
he collection, which measures 2.5 cm in diameter, pneumonia. Her course is complicated by respiratory
is lanced, producing a small amount of purulent failure requiring ongoing mechanical ventilation. She is
material, which is drained and swabbed for culture. treated with levoloxacin, piperacillin-tazobactam, and
he culture of the wound later grows Staphylococcus vancomycin, and her respiratory status improves over
aureus. the irst 2 days of admission. Her medical history is
In addition to excellent wound care, which of the notable for type 2 diabetes, hypertension, and end-stage
following antibiotics would be the best choices for out- renal disease. She has been on hemodialysis for the last
patient treatment of this skin and soft tissue infection? 7 weeks through a tunneled catheter while a istula in
A. Levoloxacin the right upper extremity matures. On hospital day 4
B. Clarithromycin she redevelops daily fevers as high as 102.5°F and hemo-
C. Trimethoprim-sulfamethoxazole dynamic instability that persists for 2 days. On hospital
D. Penicillin day 6, blood cultures are reported as growing yeast.
E. Rifampin he best management in addition to repeating
blood cultures is:
12. A 26-year-old graduate student presents for evaluation A. Start intravenous luconazole 800 mg daily.
after being bitten on the right shin by her neighbor’s B. Remove the tunneled dialysis catheter, stop unnec-
playful Labrador puppy. Her past medical history is essary antibiotics, and observe.
notable for undergoing splenectomy for treatment C. Start intravenous isavuconazonium.
of idiopathic thrombocytopenic purpura at age 24, D. Start intravenous caspofungin.
which was curative. She takes no medications and has E. Start intravenous liposomal amphotericin.
no allergies. On examination she has normal vital signs
and appears well. On the right shin are two puncture 15. A 43-year-old woman calls to report that she is very
marks, which are still bleeding slightly. here is no concerned because she found a large lump under
purulence and no surrounding erythema of the skin. her left arm. She has no personal or family history
he most appropriate management is: of malignancy. She has no other complaints. On
A. Sequester the puppy and treat the patient with examination, her vital signs are normal. She has one
human rabies immunoglobulin. tender 2 × 3 cm mobile mass under her left arm,
B. Oral trimethoprim-sulfamethoxazole with overlying erythema. She works at an urban ani-
C. No therapy is necessary. mal shelter. At home she has two recently adopted
D. Intravenous ceftriaxone kittens, one of whom was ill about 2 months earlier.
E. Oral amoxicillin-clavulanate She tried to force-feed the kitten antibiotics and was
scratched repeatedly.
13. A 67-year-old female is recovering following an elective Among cat-associated zoonoses, the most likely
total hip arthroplasty. On postoperative day 5, she com- pathogen in this case is:
plains of worsening diarrhea and abdominal pain. Her A. Bartonella henselae
white blood cell count has risen from 9000 to 21,000 B. Toxocara cati
cells/µL. Testing of the stool reveals a positive glutamate C. Toxoplasma gondii
dehydrogenase antigen and a positive toxin A/B assay D. Yersinia pestis
conirming the diagnosis of Clostridium diicile. he E. Pasteurella multocida
4 C HA P T E R 1 Infectious Diseases

16. A 26-year-old woman presents for a new primary care of 527 mg/dL. Urinalysis is remarkable for ketones and
provider visit. She has no health complaints, and her glucose. CT of the sinuses reveals opaciication of the
medical history is notable only for appendectomy at left frontal, sphenoid, and ethmoid sinuses. Endoscopic
age 23. She immigrated to the United States from evaluation by an otolaryngologist reveals a black eschar
Vietnam 7 years ago. She recently inished her under- over the left middle turbinate.
graduate degree and will start a new job as a irst-grade In addition to treatment with insulin for diabetic
teacher in a few weeks. She is sexually active with her ketoacidosis, appropriate management includes:
boyfriend of 1 month and reports using condoms for A. Voriconazole
birth control. She has a history of occasionally smok- B. Lipid formulation of amphotericin B
ing marijuana but has not done so for over 3 years. C. Caspofungin
In addition to giving inluenza and tetanus/diphthe- D. Fluconazole
ria/acellular pertussis vaccines, screening for hepatitis B E. Cefepime
infection is ordered. Which detail in the patient’s history
indicates that she should be screened for hepatitis B? 19. An obese 57-year-old man with asthma who smokes
A. New sexual partner two packs per day of cigarettes is admitted with 3
B. Woman of childbearing age days of fever, nonproductive cough, and shortness of
C. History of marijuana use breath. Home medications include luticasone inhaler
D. Country of birth twice a day and albuterol inhaler as needed for wheez-
E. New profession as a teacher ing. Oxygen saturation is 94% on 2 L of oxygen by
nasal cannula. Chest x-ray shows dense multifocal
17. A 45-year-old woman was diagnosed with HIV infec- consolidations. He is admitted to the medical service
tion (224 CD4 T cells/µm3) and smear-negative, cul- and treated with levoloxacin. he following labora-
ture-positive pulmonary tuberculosis after presenting tory indings are sent:
with chronic cough. Chest CT showed left lower lobe Urine legionella antigen: positive
interstitial iniltrates. Rapid tuberculosis drug suscepti- Urine streptococcal antigen: negative
bility testing showed no evidence of drug resistance. he Expectorated sputum Gram stain: no polys, 1+ epi-
patient was started on isoniazid, rifampin, ethambutol, thelial cells
and pyrazinamide. After 2 weeks, the patient was started Expectorated sputum bacterial culture: 2+ oral lora,
on antiretroviral therapy with emtricitabine/tenofovir 1+ Candida albicans
and dolutegravir. Her cough initially improved but Based on these results the best treatment plan is:
then worsened after about 1 month of tuberculosis A. Stop levoloxacin and add micafungin.
treatment. She also developed progressive shortness of B. Stop levoloxacin; start azithromycin and mica-
breath. A chest x-ray showed new extensive right upper fungin.
lobe opacities. Sputum smear microscopy was negative. C. Continue levoloxacin and add luconazole.
he patient should now: D. Continue levoloxacin, add luconazole, and pur-
A. Switch to an empiric regimen for treatment of sue bronchoscopy.
multidrug-resistant tuberculosis. E. Continue levoloxacin and observe.
B. Initiate systemic corticosteroids to control symp-
toms of paradoxical tuberculosis immune recon- 20. A 25-year-old woman presents to the emergency
stitution inlammatory syndrome (IRIS). department with fever and back pain. he patient
C. Enroll in directly observed tuberculosis treatment has been using intravenous heroin for the past few
given the high likelihood of poor adherence to years; she had one prior episode of soft tissue abscess
therapy. after injection but no other illnesses in the past. She
D. Begin trimethoprim/sulfamethoxazole for treat- now complains of 2 weeks of fevers, sweats, muscle
ment of Pneumocystis jirovecii infection. aches, and some low back pain. On examination she
E. Proceed to open lung biopsy. is tachycardic, diaphoretic, febrile (102°F), and ill
appearing. Cardiac examination reveals a new sys-
18. A 45-year-old man presents to the emergency room with tolic murmur. Blood is drawn for basic laboratory
2 days of severe headache over the left eye and fevers. He indings and blood cultures (two sets). Given her ill
reports nearly constant tearing of the left eye and tender- appearance, the admitting physician decides to start
ness over the left eye for 24 hours. His medical history empiric antibiotics for the most likely pathogens
is notable for type 1 diabetes mellitus complicated by immediately.
peripheral neuropathy and chronic kidney disease. His he best empiric antibiotic regimen for this patient is:
examination is notable for an ill-appearing middle-aged A. Vancomycin + cefepime
man with chemosis of the left eye and periorbital and B. Vancomycin + gentamicin + rifampin
palpebral erythema extending over the left side of the C. Vancomycin + caspofungin
nose. Laboratory data are notable for a blood glucose D. Ampicillin + gentamicin
CHAPTER 1 Infectious Diseases 5

21. A 47-year-old male with quadriplegia secondary to in bilateral cervical and groin distribution. He
a motor vehicle accident as a young adult presents has a maculopapular rash over his chest and back.
with fever, fatigue, and foul-smelling urine. He has Laboratory data: WBC 2200/µL, hematocrit 30%,
a chronic indwelling Foley catheter due to urinary platelets 103,000/µL, blood urea nitrogen (BUN)
retention, and he reports multiple hospitalizations 20 mg/dL, creatinine 1.0 mg/dL, AST 66/µL, ALT
for catheter-associated urinary tract infections. On 72/µL, alkaline phosphatase 120/µL, total bilirubin
arrival to the emergency department, he is noted to 0.9 mg/dL. Blood cultures are drawn.
be confused with a temperature of 102.3°F, a respira- Which of the following is the most appropriate
tory rate of 23 breaths per minute, a heart rate of 116 panel of tests to order next?
beats per minute, and a blood pressure of 75/43 mm A. Hepatitis A, B, and C serologies
Hg. Urinalysis inds 3+ leukocyte esterase and positive B. Urine gonorrhea and chlamydia probes, syphilis
nitrite with microscopy revealing 100–200 WBC per serology
high-powered ield (HPF) with 3+ bacteria. Blood and C. Rheumatoid factor (RF), echocardiogram
urine cultures are obtained and laboratory indings are D. Blood smear, lactate dehydrogenase (LDH), bone
pending. marrow biopsy
In addition to administering broad-spectrum anti- E. Blood smear, Epstein-Barr virus (EBV) serologies,
biotics, the next best step is: CMV serologies, HIV 1/2 antigen/antibody, HIV
A. Initiation of dopamine as a vasopressor RNA
B. Administration of intravenous hydrocortisone
C. Administration of 30 mL/kg of crystalloid 24. A 48-year-old male presents to the emergency
D. Initiation of norepinephrine as a vasopressor department complaining of shortness of breath wors-
ening over the past 2 days and weakness. Admission
22. A 23-year-old woman with acute myeloid leukemia chest x-ray reveals pulmonary edema, and laboratory
(AML) who underwent standard induction chemo- indings are notable for potassium 6.8 mmol/L. On
therapy with cytarabine and daunorubicin 11 days review of his past medical history, you learn that he
ago has recurrent fever to 103.1°F associated with mal- has severe type 1 diabetes mellitus complicated by
aise and sweats. She also endorses a sore mouth and nephropathy for which he has been on peritoneal
throat as well as mild diarrhea, which she attributes to dialysis for the past year. He reports 100% compli-
her recent chemotherapy. Her laboratory indings are ance with his peritoneal dialysis regimen at home.
notable for an absolute neutrophil count of 28. She Two months ago he was admitted for methicillin-
irst developed fever and neutropenia 8 days ago and resistant Staphylococcus aureus (MRSA) bacteremia
was treated with empiric cefepime. Her fevers resolved related to an infected foot ulcer for which he was
within 24 hours, and she remained afebrile until 2 treated with a course of intravenous vancomycin. He
hours ago. When fevers redeveloped, blood cultures is admitted for acute management of hyperkalemia
were drawn. and to transition to hemodialysis. On hospital day 4,
Which is the most appropriate antimicrobial agent one day after surgery to create an arteriovenous is-
to add? tula, he reports an increasingly productive cough and
A. Fluconazole spikes a fever to 101.1°F. A repeat chest x-ray reveals
B. Vancomycin a new consolidative opacity in the right lung.
C. Caspofungin Based on clinical and radiologic evidence suggest-
D. Meropenem ing pneumonia, you recommend:
E. Daptomycin A. Levoloxacin
B. Ertapenem + vancomycin
23. A 22-year-old man presents to his primary care C. Piperacillin/tazobactam + amikacin
physician’s office with fever and sore throat. The D. Imipenem + ciproloxacin
patient was feeling well until 1 week ago when he E. Cefepime + levoloxacin + vancomycin
developed fever, malaise, fatigue, and sore throat;
he later developed diarrhea. He has lost 5 pounds 25. A 21-year-old college student with moderate to
in the past month. The patient is a student at a local severe asthma presents to the student health cen-
college, he does not smoke, he drinks alcohol a few ter with a sore mouth for several days. hough it
times per week, and he does not use any recreational is bothersome, it has not interfered with his eating
drugs. He is sexually active with men and has had or drinking. He denies odynophagia. Ten days ago
two new male partners over the past few months. he was hospitalized for 2 days with a severe asthma
He is up to date on all of his vaccinations. On lare. He was treated with a 2-week prednisone taper
examination he is thin, febrile, and mildly tachy- for his asthma and with a 5-day course of azithro-
cardic. His oropharynx is erythematous without mycin for possible respiratory tract infection. His
exudates; he has palpable small lymphadenopathy regular medications include inhaled salmeterol and
6 C HA P T E R 1 Infectious Diseases

luticasone. On examination he is a relatively well- Based on identiication of bacteria grown in blood


appearing young man with white curd-like plaques culture, the suspicion for endocarditis diminishes
adherent to the soft and hard palate. signiicantly.
In addition to education about rinsing the mouth Which of the following bacteria grew in culture?
after using a steroid inhaler and considering HIV test- A. Haemophilus parainluenzae
ing, the next best step is: B. Eikenella corrodens
A. Clotrimazole troches C. Streptococcus gallolyticus
B. Chlorhexidine mouthwash D. Bacteroides fragilis
C. Referral to oral medicine for biopsy E. Enterococcus faecalis
D. Oral posaconazole
E. Single-dose intravenous micafungin 29. A 28-year-old woman presents to the emergency
department with fever, malaise, abdominal pain, nau-
26. A 26-year-old female is hospitalized in a burn unit sea, and vomiting. Initial laboratory indings show
after sufering deep burns over 55% of her body in WBC 1800/µL, hematocrit 27%, platelets 100,000/µL,
a house ire. She remains intubated with access via AST 170/UL, ALT 195/UL, alkaline phosphatase 422
central venous catheter. On hospital day 10, the IU/L, and total bilirubin 2.1 mg/dL. A CT scan of the
microbiology lab calls with the result of a blood cul- abdomen and pelvis reveals difuse lymphadenopathy
ture that was sent in the setting of a fever to 102°F. and hepatosplenomegaly. Further diagnostics reveal
he lab reports that Acinetobacter baumannii is that HIV antibody is positive with a CD4 count of
growing in the blood, which is testing positive for 18/µL (4% of total lymphocytes). Biopsy of a lymph
extended-spectrum beta-lactamases and a carbapen- node reveals numerous organisms on acid-fast stain-
emase. ing. PCR of the sample with numerous organisms is
Based on these results, which antibiotic is most consistent with mycobacterium avium intracellulare (or
likely to be efective against this pathogen? mycobacterium avium complex [MAC]). he patient is
A. Meropenem started on treatment with clarithromycin and ethambu-
B. Colistimethate tol while awaiting the results of resistance testing from
C. Ciproloxacin mycobacterial isolator blood cultures.
D. Piperacillin/tazobactam When is the best time to start antiretroviral therapy
E. Gentamicin for this patient?
A. Start three antiretroviral drugs in a staggered fash-
27. A 37-year-old woman with HIV (CD4 count 523/µL, ion, adding one every 2 weeks over the next 6
HIV viral load <20/µL), asthma, and allergic rhinitis weeks.
presents to her primary care physician oice com- B. A few days after initiation of clarithromycin and
plaining of worsening symptoms of allergic rhinitis. ethambutol, but within 2 weeks
She has had worsening rhinorrhea, itchy and watery C. After 2 weeks on clarithromycin and ethambutol,
eyes, and dry cough over the past few weeks this if no side efects
spring. Her current medications are tenofovir, emtric- D. After completion of 6 weeks of clarithromycin and
itabine, darunavir, ritonavir, loratadine, and inject- ethambutol, to decrease risk of immune reconsti-
able medroxyprogesterone. She requests an additional tution inlammatory syndrome (IRIS)
medication for control of her allergic rhinitis symp- E. After the patient is discharged from the hospital,
toms. in the outpatient setting with documented patient
Which of the following drugs should not be pre- capacity for adherence to follow-up and medica-
scribed because of a potentially harmful drug–drug tions
interaction?
A. Oral cetirizine 30. A 45-year-old man with a long-standing history of well-
B. Inhaled albuterol controlled HIV presents to his primary care physician
C. Ophthalmic nedocromil for a routine visit. He is feeling well, with no symp-
D. Inhaled luticasone tomatic complaints. He reports 100% adherence to his
E. Oral montelukast antiretroviral regimen: tenofovir, emtricitabine, and ril-
pivirine. He works in real estate and lives with his hus-
28. A 48-year-old man presents with 4 days of fevers, band and their dog. He smokes approximately one pack
night sweats, and malaise. He has a history of bicus- of cigarettes per day, as he has for the past 20 years. He
pid aortic valve with aortic regurgitation. On exami- drinks six alcoholic beverages per week, and he does not
nation he is febrile (102.4°F) and tachycardic. Blood use recreational drugs. His family history is notable for
cultures are drawn. A transthoracic echocardiogram coronary artery disease in both parents but no cancer in
does not show any valvular vegetations or abscess. irst-degree relatives. Vital signs: heart rate 82 beats per
CHAPTER 1 Infectious Diseases 7

minute, blood pressure 137/86 mm Hg, BMI 32 kg/ and fatigue. he patient has a history of childhood
m2, and physical examination is unremarkable. Labora- asthma, but she has not used inhalers or other medi-
tory indings show CD4 count 470/µL, HIV viral load cations for asthma in several years. She is otherwise
is <20/µL (undetectable), and complete blood count healthy, and she takes no medications. She has a his-
(CBC) and chem-20 are normal. You plan to continue tory of a rash reaction to clarithromycin. On exami-
his current antiretroviral regimen, and you discuss addi- nation, her temperature is 99.0°F, heart rate is 92
tional health care maintenance eforts with him. beats per minute, blood pressure is 132/85 mm Hg,
Given his history, which of the following is the respiratory rate is 18 breaths per minute, and oxy-
most important health care maintenance item to pur- gen saturation is 96% on room air. She appears
sue during this visit? mildly uncomfortable and is coughing during the
A. Start sulfamethoxazole/trimethoprim for prophy- examination; she has no lesions in the oropharynx,
laxis against Pneumocystis pneumonia. sclerae are clear, maxillary sinuses are mildly tender
B. Counsel patient to decrease alcohol intake, and on percussion, there is no nasal discharge/drainage,
refer for alcohol dependence treatment. tympanic membranes are clear, and she has a few
C. Counsel patient to quit smoking, and discuss small (<1.5 cm) palpable cervical lymph nodes. Her
medications and supports for smoking cessation. lungs are clear except for occasional faint expiratory
D. Screen for toxoplasma serostatus to determine risk wheezes.
for toxoplasma reactivation in the future. he most appropriate management is:
E. Refer for early colon cancer screening by colonos- A. Oral moxiloxacin, inhaled albuterol, and intrana-
copy. sal oxymetazoline
B. Oral amoxicillin-clavulanate, inhaled luticasone,
31. An 84-year-old man with a history of coronary artery and saline nasal irrigation
disease, diabetes mellitus, and chronic renal insui- C. Oral dextromethorphan, inhaled albuterol, and
ciency presents to the emergency room with progres- intranasal ipratropium
sive headache, fevers, and neck pain. he patient was D. Intramuscular ceftriaxone, oral pseudoephedrine,
in his usual state of health until 4 days prior when the and inhaled salmeterol
staf and his friends at his assisted living facility noted E. Intramuscular inluenza vaccine, oral guaifenesin,
he started complaining of feeling ill with headache and and inhaled tiotropium
nausea. On the morning of presentation he was found
in his room confused. In the emergency room he was 33. A 29-year-old woman who is otherwise healthy pres-
febrile and confused, and became somnolent during ents for a routine prenatal visit at 14 weeks’ gestational
his care there. Head CT showed no acute processes. A age. She is feeling well and has no symptomatic com-
lumbar puncture was performed, cerebrospinal luid plaints, and physical examination is consistent with
(CSF) examination showed glucose 22 mg/dL (serum normal pregnancy, otherwise unremarkable. She has
glucose 112 mg/dL), protein 97 mg/dL, red blood routine prenatal laboratory indings checked, which
cell (RBC) 7, WBC 489 with 87% neutrophils, 7% show the following results: hemoglobin 11.2 g/dL,
monocytes, and 6% lymphocytes. CSF is sent to the rubella IgG positive, HIV-1/2 antigen/antibody nega-
microbiology lab for Gram stain and culture. tive, treponemal IgG (by enzyme immunoassay [EIA])
Which of the following is the most appropriate positive. Follow-up rapid plasma reagin (RPR) is also
empiric antibiotic regimen to initiate while awaiting positive, with a titer of 1:16, and FTA-ABS is also pos-
the results of the CSF Gram stain and culture? itive. he patient has never had prior syphilis testing.
A. Vancomycin, ceftriaxone, ampicillin She reports a history of severe allergy to penicillin with
B. Vancomycin, cefepime, acyclovir a “feeling of throat closing.”
C. Vancomycin, meropenem he most appropriate management for this patient is:
D. Ampicillin, gentamicin, acyclovir A. No treatment now due to risk of toxicity; follow
E. Ampicillin, trimethoprim-sulfamethoxazole, amp- clinically and repeat syphilis testing at 20 weeks’
hotericin B gestational age
B. Treat with doxycycline 100 mg orally twice daily
32. A 27-year-old woman presents to her primary care for a 21-day course.
physician complaining of fever, cough, sinus pres- C. Treat with ceftriaxone 1 g IM once daily for 10
sure, and malaise. She reports onset of symptoms 5 days.
days ago, with fevers for the irst 2 days with tem- D. Treat with azithromycin 2 g orally in a single dose.
peratures as high as 101.1°F. Highest temperature in E. Allergy consultation and admission for desensiti-
the past 3 days has been 99.8°F. She reports ongo- zation to penicillin in order to facilitate treatment
ing symptoms of frequent cough productive of scant with benzathine penicillin G 2.4 million units IM
white sputum, nasal congestion, mild sinus pressure, once weekly for 3 weeks
8 C HA P T E R 1 Infectious Diseases

34. A 24-year-old man with well-controlled HIV presents and on sinonasal examination he has septal perfo-
for routine follow-up primary care visit and notes some ration with no obvious exudates or other abnor-
dysuria for the past several days. Physical examination malities. Blood tests show white blood cell count of
is unremarkable. Urinalysis shows 10 WBC, no epi- 4200/µL, hemoglobin 9.1 g/dL, and creatinine 2.2
thelial cells, 2 RBC, no bacteria per HPF. Urine testing mg/dL. Urinalysis shows too numerous to count red
for gonorrhea is positive by nucleic acid ampliication blood cells. Chest x-ray shows some abnormal opac-
test (NAAT) probe, and urine chlamydia NAAT probe ities, so chest CT is obtained, which shows multiple
is negative. he patient has no known drug allergies. pulmonary nodules.
Which of the following is the recommended treat- he test most likely to suggest the diagnosis in this
ment regimen? case is:
A. Levoloxacin 500 mg orally once daily for 7 days + A. Sputum smear for acid-fast bacilli (AFB)
doxycycline 100 mg orally twice daily for 7 days B. AFB smear of biopsy of a pulmonary nodule
B. Ceftriaxone 250 mg IM single dose C. Serum test for antineutrophilic cytoplasmic anti-
C. Ceixime 400 mg orally single dose + doxycycline bodies (ANCA)
100 mg orally twice daily for 7 days D. Serum test for galactomannan
D. Ceftriaxone 250 mg IM single dose + azithromy- E. Serum interferon-gamma release assay (IGRA)
cin 1 g orally single dose
37. A 62-year-old woman with multiple sclerosis and a
35. A 32-year-old man presents to his primary care phy- neurogenic bladder now has a chronic indwelling uri-
sician complaining of painful perianal lesions. he nary catheter, after failing management with intermit-
patient is sexually active with multiple male partners. tent use of urinary straight catheters. She presents for
He has had negative screening for sexually transmitted a routine primary care visit and has no current symp-
diseases, including HIV, gonorrhea, and chlamydia, in tomatic complaints. She is interested in discussing
the past (his last screening tests were 6 months ago). strategies to decrease the risk of urinary tract infec-
On physical examination he is overall well appearing, tions in the future. In the past when she has developed
but in the perianal area he has multiple shallow ulcer- urinary tract infections they often precipitated a wors-
ations grouped in the right perianal area. here is no ening of her multiple sclerosis, and she often requires
rectal discharge and no palpable lymphadenopathy, hospitalization, so she hopes to prevent the need for
but the ulcerations are tender and painful even when hospitalization in the future.
not palpated. Which of the following strategies would be most
he type of diagnostic test that is most likely to successful at achieving her goals?
conirm the diagnosis of this active condition is: A. Monitoring for early signs and symptoms of uri-
A. Bacterial culture of a swab of the ulcers nary tract infection, with expedited early urinaly-
B. Viral culture of a swab of the ulcers sis, urine culture, and empiric treatment while
C. Urine NAAT probe awaiting culture results when symptoms develop
D. Blood serologic test B. Routine screening with urinalysis for pyuria at
E. Urinary antigen test regular intervals with early empiric treatment for
urinary tract infection if pyuria is detected, even in
36. A 47-year-old man presents to the emergency depart- the absence of symptoms
ment with complaint of hemoptysis. He was born C. Addition of gentamicin solution to the catheter
and raised in upstate New York, currently manages drainage bag at regular intervals
a restaurant and bar that he and his siblings own, D. Chronic prophylaxis with methenamine salts to
and is married with two children. His only travel decrease bacteria
in the last few years was a trip to Montreal. He has E. Chronic prophylaxis with ciproloxacin to decrease
been healthy until approximately 6 months prior bacteria and infections
when he developed sinusitis. He has been treated by
his primary care physician with courses of amoxi- 38. A 45-year-old man with a history of prior open reduc-
cillin, amoxicillin-clavulanate, and moxiloxacin tion and internal ixation (ORIF) of a left femur frac-
for episodes of sinusitis over the past few months, ture in the past now presents with his third episode
but his symptoms persist. He has also had several of cellulitis in the left leg. He was well until 1 day
episodes of epistaxis in the last few months. Over prior when he developed sudden onset of malaise,
the past 2 weeks he developed a cough, which was fever, nausea, and erythema and pain in the left leg.
initially nonproductive, but during the past 2 days He presented to the emergency room overnight and
he had a few episodes of hemoptysis. On physical was treated empirically with vancomycin overnight.
examination he has lost 5 lb since his last exami- He improves gradually overnight. In discussion the
nation 1 month prior, he is thin but comfortable, following day he asks whether there are any strategies
CHAPTER 1 Infectious Diseases 9

to decrease the frequency of his episodes of cellulitis in and takes no medications. She lives in New Hamp-
the future. shire and spends time walking her dog in the woods
Which of the following antibiotics, when taken regu- frequently, but she does not remember any speciic
larly as prophylaxis, has been shown to decrease the inci- tick bites. Approximately 10 days earlier she developed
dence of cellulitis among patients with recurrent cellulitis? upper respiratory infection (URI) symptoms, which
A. Sulfamethoxazole-trimethoprim improved over a few days and then resolved after 7
B. Doxycycline days. Over the past 3–4 days she developed right ear
C. Clarithromycin pain. On physical examination she is noted to have a
D. Levoloxacin right facial droop and some lesions with serous drain-
E. Penicillin age in the right external ear canal. Her mucous mem-
branes are dry.
39. A 32-year-old woman presents for routine prenatal Which of the following is the most appropriate
care at 12 weeks’ gestational age. She is taking prenatal treatment for this patient?
vitamins and is feeling well. She reports that a friend A. Doxycycline 100 mg orally twice daily for 14 days
recently gave birth to a daughter who was diagnosed B. Valacyclovir 1000 mg orally three times daily for
with congenital toxoplasmosis, and she would like to 14 days
know how to prevent this infection during her preg- C. Ciproloxacin otic solution to the right ear 4 times
nancy. She is a kindergarten teacher. She lives with her per day for 7 days
spouse, who cares for their 10-year-old indoor cat. She D. Ciproloxacin otic solution + amoxicillin-clavula-
likes to garden in her free time. nate 875/125 mg orally twice daily for 10 days
In addition to washing fruits and vegetables before E. Prednisone 60 mg orally once daily for 5 days
eating, which of the following lifestyle changes is rec-
ommended to reduce risk for acute toxoplasma infec- 42. A 26-year-old man presents with dysuria, which has
tion during pregnancy? been persistent for more than 1 week. Urinalysis shows
A. Cook meat to “well done.” 12 WBC, 1 RBC, and no bacteria per HPF, and the
B. Avoid ingestion of pork or any pork products. urine culture is negative. Urine NAAT probes for chla-
C. Give the cat up for adoption. mydia and gonorrhea are negative. Serum testing for
D. Stop gardening. HIV 1/2 antigen/antibodies is negative.
E. Take leave from work at the start of the third tri- Which of the following organism is the most likely
mester to avoid transmission from her students. cause of the patient’s symptoms?
A. Trichomonas vaginalis
40. A 74-year-old man with poorly controlled diabetes, cor- B. Herpes simplex virus (HSV)
onary artery disease, end-stage renal disease, and periph- C. Mycoplasma genitalium
eral vascular disease presents with pain at a chronic foot D. Haemophilus ducreyi
ulcer site. he patient has had a nonhealing ulcer on his E. E. coli
left great toe for several months but no other symptoms.
Over the past few days he developed purulent drain- 43. A 71-year-old woman with hypertension develops tem-
age from the ulcer bed, as well as erythema, pain, and poral headaches, fevers, and weight loss. She is diagnosed
swelling of the toe, which is now tracking up the foot. with giant cell arteritis by temporal artery biopsy and
he margins of the toe ulcer have also started to turn starts treatment with prednisone 50 mg per day. Her
black. He has a fever of 100.7°F at the time of presen- symptoms improve markedly, and the prednisone dose
tation. Laboratory indings reveal a white blood cell is slowly tapered starting 3 weeks later. Ten weeks after
count of 13,500 cells/µL. Blood cultures are sent, and starting the prednisone taper she develops fevers in the
debridement—during which cultures of the base will be 100–101°F range despite taking 20 mg of prednisone
obtained—is planned for later in the day. per day. She also notes a dry cough. She is treated with a
Which of the following antibiotic regimens would course of azithromycin but continues to have a cough and
be appropriate initial therapy for this patient while fevers. She also notes dyspnea on exertion. She presents to
awaiting debridement and culture results? the emergency department, where she is found to have a
A. Vancomycin temperature of 100.6°F, respiratory rate of 38 breaths per
B. Cefazolin and metronidazole minute, and oxygen saturation of 92% while breathing
C. Ampicillin-sulbactam ambient air. On exam she has no jugular venous disten-
D. Vancomycin and piperacillin-tazobactam tion or peripheral edema. She has faint bilateral crackles
E. Ertapenem in both lungs. A chest x-ray shows bilateral interstitial
iniltrates. Serum β-d-glucan is >500 pg/mL.
41. A 67-year-old woman presents to the emergency room In addition to treatment with antibiotics for com-
with right ear pain. She is otherwise healthy at baseline munity-acquired pneumonia, which of the following
10 C HA P T E R 1 Infectious Diseases

treatments is most appropriate additional empirical Staphylococcus aureus (MRSA). he patient is treated
therapy? with vancomycin while an inpatient, and his fevers
A. Ivermectin resolve. He is then discharged with oral linezolid for
B. Trimethoprim-sulfamethoxazole another 10 days. After 7 days he returns to the emer-
C. High-dose steroids gency room complaining of recurrent fever and mal-
D. Furosemide aise. His temperature is 103.7°F, heart rate is 116 beats
E. heophylline per minute, blood pressure is 180/92 mm Hg, and
oxygen saturation is 98% on room air. On examina-
44. A 22-year-old man presents to primary care physi- tion he is somewhat agitated and unable to sit still.
cian for routine follow-up. He is sexually active with Lungs are clear to auscultation bilaterally, there are no
men, with three partners in the past year. He had a murmurs on cardiac exam, abdomen is soft and non-
recent urgent care visit for urethritis, was diagnosed tender, and the lower left leg prior incision and drain-
with gonorrhea, and was treated with ceftriaxone and age site is healing.
azithromycin. He is now feeling well, with no cur- he most likely cause of this patient’s new symp-
rent symptomatic complaints. he physician focuses toms is:
this routine visit on sexually transmitted infections A. Drug–drug interaction
(STIs), including risk-reduction counseling, conirm- B. Recurrent MRSA abscess
ing completion of HPV-vaccine series, distribution of C. MRSA bacteremia due to endocarditis
condoms, and discussion of preexposure prophylaxis D. Hospital-acquired pneumonia
(PrEP) for HIV prevention. he patient is interested E. Clostridium diicile colitis
in PrEP and inquires about the usual treatment plan
and the risks and beneits associated with PrEP. 46. A 53-year-old man with a prior history of idiopathic
Which of the following statements about antiretro- thrombocytopenic purpura treated with a course of
viral PrEP is true? steroids, rituximab, and ultimately splenectomy 4
A. HIV screening with combination antigen/anti- months ago now presents with fever and chills. he
body test should be performed at baseline before patient spent a week on Nantucket for a summer vaca-
initiation of PrEP and annually while on PrEP. tion and was feeling well until 2 days after he returned
B. Taking tenofovir/emtricitabine once daily with from vacation, when he developed fevers as high as
excellent adherence for PrEP can decrease risk of 103.2°F, shaking chills, and headache. He presented
HIV acquisition by about 25%. to a local hospital, where initial complete blood count
C. Patients taking PrEP should have regular screening showed WBC 12,000/µL, hemoglobin 8.2 g/dL,
for other STIs at least every 6 months, including platelets 80,000/µL, normal electrolytes, serum creati-
syphilis, gonorrhea, and chlamydia. nine 2.1 mg/dL, and alkaline phosphatase 206 IU/L.
D. In the HIV uninfected population there is little A blood smear showed parasites within the red blood
risk for tenofovir nephrotoxicity while taking cells; the parasitemia burden was assessed as 12%.
PrEP, so there is no need for monitoring of renal Which of the following is the most appropriate
function. treatment regimen for this patient at this point?
E. In most populations the risks of PrEP outweigh A. Azithromycin and atovaquone/proguanil
the beneits because patients who are prescribed B. Quinine and clindamycin and consideration of
PrEP have increased sexual risk behaviors and red cell exchange transfusion
increased incidence of STIs, including HIV when C. Ceftriaxone and doxycycline
nonadherent to PrEP. D. High-dose corticosteroids, intravenous immuno-
globulin, and initiation of plasmapheresis
45. A 55-year-old man with a history of hypertension, E. Intravenous artesunate
coronary artery disease, and depression presents with
a soft tissue infection on the leg. He has no prior his- 47. A 36-year-old man with psoriasis, for which he takes
tory of soft tissue infections. His current medications methotrexate, traveled to Arizona for 2 weeks for a
are lisinopril, clopidogrel, aspirin, atorvastatin, and family reunion and developed a fever 2 days before
duloxetine. He has no known medication allergies. On returning to his home in New York. He was feeling
examination, he has a fever (temperature is 101.2°F) well during the trip and enjoyed the irst week of the
but vital signs are otherwise normal. here is an area reunion. He bunked with extended family, including
of erythema on the lower left leg, originating from a small children and two dogs. He ate food at the hotel
punctate wound where the patient states he sustained a and drank primarily bottled water. He went hiking in
spider bite. he area is warm, swollen, and tender, and the desert on three occasions. He swam in the hotel
there is a central 4-cm area of luctuance. Incision and pool but engaged in no fresh water swimming. he
drainage of the abscess is performed at the bedside. weather was dry throughout the trip and very windy at
A culture of drained pus grows methicillin-resistant times. Two days before returning home he developed
CHAPTER 1 Infectious Diseases 11

fever, fatigue, malaise, dry cough, and chest pain. He 49. A 53-year-old woman who is otherwise healthy devel-
took acetaminophen and rested, then returned home ops fever, headache, malaise, and then cough, which
to New York. After 10 days the symptoms had not is persistent and worsens over a couple of days. he
signiicantly improved, so he presented to his primary patient lives in Missouri, where she works on a dairy
care provider. Physical examination was remarkable farm and also tends sheep for wool as an additional
only for low-grade fever. Chest x-ray showed subtle source of income. She spends her spare time hunting
left hilar iniltrate and lymphadenopathy. He was deer and rabbits. She is married and is monogamous
treated with a 5-day course of azithromycin with no with her husband. At the time of presentation she was
change in his symptoms. mildly hypoxic and febrile, and her condition rapidly
he best diagnostic test to send at this point would be: worsened, ultimately requiring mechanical ventila-
A. Blood serologic test for Coccidioides tion. Chest imaging showed multifocal iniltrates and
B. Serum 1,3-β-d-glucan testing progressive pleural efusions, as well as hilar lymph-
C. Urinary legionella antigen adenopathy. At the time of admission her laboratory
D. Biopsy of the hilar lymph node for fungal culture indings were normal with the exception of WBC
E. Serum galactomannan 14,000/µL. However, she developed progressive renal
failure and abnormal liver function tests over the irst
48. A 31-year-old man with a prosthetic aortic valve 2 days after admission. Blood, urine, and sputum cul-
presents with fevers. He had his aortic valve replaced tures are all negative repeatedly. She has been treated
with a mechanical prosthesis at age 24 due to con- with vancomycin, cefepime, and metronidazole with
genital bicuspid aortic valve, and he has been doing no improvement.
well since then. His medications include warfarin he most likely etiologic organism for her current
and lisinopril. He has no known drug allergies. At condition is:
the time of admission to the hospital, blood cultures A. Tropheryma whipplei
are positive for MRSA. After extensive evaluation, no B. Babesia microti
source for the bacteremia is identiied. He is treated C. Borrelia lonestari
for presumed prosthetic valve endocarditis, even in D. Francisella tularensis
the absence of suggestive indings on transesophageal E. Anaplasma phagocytophilum
echocardiogram. His antibiotic regimen is vancomy-
cin, gentamicin, and rifampin. After 3 days the blood 50. A 22-year-old woman who is healthy at baseline sus-
cultures clear of bacteria, he clinically improves, and tains minor blunt trauma to the right thigh after she
he is eventually discharged to a rehab facility to bumps her leg on the edge of a table. Within hours
complete a course of vancomycin, gentamicin, and after the bump she develops fever and severe right leg
rifampin. After 3 weeks he returns to the emergency pain such that she is barely able to walk. She presents
room with an acute stroke, which appears embolic on to a local emergency room, where she is febrile and
MRI/MRA of the brain. Laboratory indings show hypotensive. Her right thigh appears dusky, and she
WBC 12,000/µL, hemoglobin 9.8 g/dL, platelets complains of pain tracking down to the foot and up
167,000/µL, creatinine 0.9 mg/dL, liver function to the lower abdomen. She is taken immediately to the
tests are normal, partial thromboplastin time 36.0 operating room, where operative exploration reveals
seconds, and INR 1.2. Blood cultures are negative at necrotizing myositis of the muscles of the thigh with-
48 hours of incubation. out gas formation, with necrotizing soft tissue infec-
he most likely cause of the patient’s new stroke is: tion tracking down the leg and up to the groin. Some
A. Persistent infectious vegetation on the prosthetic of the debrided tissue is sent for Gram stain and cul-
aortic valve ture to aid with choice of antibiotics.
B. Toxicity from gentamicin he most likely pathogen is:
C. Aortic valvular dysfunction due to perivalvular A. Methicillin-sensitive Staphylococcus aureus
abscess B. E. coli
D. Hypercoagulable state due to loss of gut lora while C. Group A streptococci
on antibiotics D. Methicillin-resistant S. aureus
E. Drug–drug interaction of warfarin with rifampin E. Aeromonas hydrophila

Chapter 1 Answers
1. ANSWER: E. Dengue travelers returning with fever should be evaluated for
Leptospirosis can present in many diferent ways, malaria (even in cases where they had reported tak-
including headache, muscle aches, and fever, but it ing prophylaxis), the normal hematocrit and lack of
is almost always associated with freshwater exposure, other laboratory abnormalities in this young woman
such as swimming or white-water rafting. Although all are reassuring. Typhoid can also present with only
12 C HA P T E R 1 Infectious Diseases

headache and fever, and in fact although it is caused that result directly from surgery (e.g., wound infection).
by Salmonella species. it can often cause little to no hose transplanted 1–6 months before or those recently
gastrointestinal symptoms. he marked thrombocyto- treated for rejection are at risk for opportunistic infec-
penia would be unusual for this diagnosis. Hepatitis tions (such as CMV enteritis). Opportunists like CMV
A is also a risk for travelers, but the lack of gastroin- are much less likely in those transplanted more than 6
testinal symptoms and lack of liver enzyme elevation months ago and not treated for rejection recently (as
argue against it. Dengue is the second most common in this case). Patients more than 6 months after trans-
cause of systemic febrile illness travelers returning to plant who have not been treated for recent rejection are
the United States from foreign travel (12%), after at highest risk for community-acquired pathogens, like
malaria, and dengue was the most frequently identi- Salmonella and norovirus. his patient has been in con-
ied cause of systemic febrile illness among travelers tact with a hamster, which may increase her risk for Sal-
returning from Southeast Asia. Diagnosis is generally monella infection (hamsters have been associated with
clinical, although acute and convalescent sera can be Salmonella outbreaks). Although she may also be at risk
sent. Treatment is supportive. for norovirus infection as a school teacher, Salmonella
Hagmann SH, Han PV, Staufer WM, et al. Travel-associated dis- is much more likely in this case based on her history of
ease among US residents visiting US GeoSentinel clinics after high fevers and abdominal pain along with diarrhea in
return from international travel. Fam Pract. 2014;31:678–687. the absence of nausea or vomiting. Among medications
used for posttransplant immunosuppression, mycophe-
2. ANSWER: B. Suctioning of subglottic secretions nolate mofetil commonly causes diarrhea; however, the
here are a number of strategies that can help reduce new onset after years of mycophenolate and the associ-
the risk of ventilator-associated pneumonia in an intu- ated fevers all make mycophenolate an unlikely cause of
bated patient. hese usually focus around minimizing this acute illness.
sedation, preventing aspiration, reducing colonization Fishman JA, Rubin, RH. Infection in organ-transplant recipi-
of the airway and digestive tract, and minimizing con- ents. N Engl J Med. 1998;338:1741–1751; Centers for Dis-
tamination of the ventilatory circuit. To prevent aspira- ease Control and Prevention. Outbreak of multidrug-resistant
tion, the head of the bead should be maintained in a Salmonella typhimurium associated with rodents purchased at
semirecumbent position (elevated 30–45 degrees) rather retail pet stores—United States, December 2003-October
than in a fully recumbent position. One should also use 2004. MMWR Morb Mortal Wkly Rep. 2005;54:429–433.
a cufed endotracheal tube with cuf pressure set at ≥20
cm H2O with subglottic secretion drainage. To reduce 4. ANSWER: B. Pneumococcal 13-valent conjugate
colonization of the airway and digestive tract, orotra- (PCV13) vaccine
cheal intubation is preferred to nasotracheal intubation his patient is a 24-year-old man with ulcerative
to decrease the risk of sinusitis, acid-reducing medica- colitis for which he takes immunosuppressive medi-
tions such a histamine receptor 2 (H2)–blocking agents cations. Based on his immunocompromised state he
and proton pump inhibitors should be avoided except should be vaccinated for pneumococcus with both
in patients at high risk for ulcers/gastritis, and regular the conjugated and the polysaccharide vaccines. In
oral care with an antiseptic solution (e.g., chlorhexi- an immunocompromised patient under 65 years old
dine) should be performed. Routine use of oral or intra- who has never been vaccinated for pneumococcus,
venous antibiotics for prophylaxis is not recommended. the conjugate vaccine should be given irst, followed
Finally, to minimize contamination of the ventilatory by the polysaccharide vaccine (Option D) at least 8
circuit, the tubing should only be changed when visibly weeks later. Although the patient may be due for teta-
soiled or malfunctioning. nus vaccination, unless he has a clear history that he
Klompas M, Branson R, Eichenwald EC, et al. Strategies to pre- previously received the tetanus, diphtheria, acellular
vent ventilator-associated pneumonia in acute care hospitals: pertussis (TDaP) vaccine, the TDaP vaccine should
2014 update. Infect Control Hosp Epidemiol. 2014;35:915– be given now, not the tetanus, diphtheria vaccine
936; American horacic Society; Infectious Diseases Society of (Option A). he Advisory Committee on Immuni-
America. Guidelines for the management of adults with hos- zation Practices (ACIP) recommends that all persons
pital-acquired, ventilator-associated, and healthcare-associated above age 10 should be vaccinated with TDaP once
pneumonia. Am J Respir Crit Care Med. 2005;171:388–416.
due to outbreaks of pertussis in adolescents and adults
related to waning pertussis immunity. his patient
3. ANSWER: A. Salmonellosis has no speciic indications for Haemophilus inluen-
Infection risk after solid organ transplantation zae B vaccine at this time (Option C); after standard
depends on two key factors: when the transplant occurred childhood vaccination this vaccine is typically only
and whether the patient has recently been treated for indicated in adults in speciic circumstances such as
organ rejection. Transplant recipients who underwent stem cell transplantation or splenectomy. he menin-
transplantation less than a month ago are typically at gococcal B vaccine (Option E, available in two dif-
highest risk for nosocomial infections and infections ferent formulations with diferent schedules) was irst
CHAPTER 1 Infectious Diseases 13

approved in 2014. It is currently recommended for a 7. ANSWER: C. Nitrofurantoin macrocrystals 100


select population including individuals with asplenia, mg twice daily for 5 days
those with acquired or congenital complement dei- In 2010 the Infectious Diseases Society of America
ciency, or those who were potentially exposed in an (IDSA) updated guidelines for the management of
outbreak of meningococcus type B infection. Unlike uncomplicated cystitis in premenopausal women. By
the conjugate meningococcal ACWY vaccine, it is not far the most common cause of uncomplicated cystitis
routinely recommended for young adults between the is E. coli. Recommended irst-line regimens include
age of 16 and 23, but it is a consideration. nitrofurantoin macrocrystals 100 mg orally twice daily
Kim DK, Bridges CB, Harriman KH, et al. Advisory Commit- × 5 days, trimethoprim-sulfamethoxazole double-
tee on Immunization Practices Recommended Immuniza- strength orally twice daily × 3 days, fosfomycin 3 g sin-
tion Schedule for Adults Aged 19 Years or Older—United gle dose, or pivmecillinam 400 mg twice daily × 5 days
States, 2016. MMWR Morb Mortal Wkly Rep. 2016;65:88– (not available in the United States). here is no single
90; Bennett NM, et al. MMWR. 2012;61:816–819;
best agent in this group that is superior to all others for
Folaranmi T, Rubin L, Martin SW, et al. Use of serogroup
B meningococcal vaccines in persons aged ≥10 years at
the empiric management of uncomplicated cystitis.
increased risk for serogroup B meningococcal disease: he choice between agents should be individualized
recommendations of the Advisory Committee on Immu- and based on the patient’s history, including allergies,
nization Practices, 2015. MMWR Morb Mortal Wkly Rep. local resistance patterns, drug availability, and cost.
2015;64:608–612. Trimethoprim-sulfamethoxazole is efective and inex-
pensive, but allergic reactions are more common, and
5. ANSWER: C. Chest x-ray to assess for active pul- in some US geographic areas, resistance rate among
monary disease community-acquired E. coli isolates exceeds 20% and
he patient should undergo chest x-ray to evaluate therefore it is not recommended in this type of set-
for active pulmonary tuberculosis. Radiologic disease ting. Nitrofurantoin is more expensive but efective.
can be present in the absence of symptoms. Either a Fosfomycin is not as efective and is also expensive.
tuberculin skin test (PPD) or an interferon gamma However, they both exert little “collateral damage.”
release assay alone is acceptable to test for tuberculosis his term refers to the indirect adverse efects of anti-
exposure, and a positive test result from either is suf- biotic agents, including their propensity to select
icient to diagnose latent infection. Because interferon drug-resistant organisms and promote colonization
gamma release assays are not afected by previous BCG and infection with multidrug-resistant bacteria. A
vaccine, this is the preferred screening test for patients 3-day regimen of luoroquinolones such as ciproloxa-
who have previously received this vaccine. Sputum cin is highly eicacious in the management of uncom-
evaluation should only be pursued if the patient has plicated cystitis. However, luoroquinolones are no
concerning symptoms or signs of active disease on longer recommended as irst-line empiric agents due
chest imaging. he Centers for Disease Control and to their potential to impact gastrointestinal lora and
Prevention (CDC) web site provides updated infor- also for potential side efects. Beta-lactam antibiotics,
mation about diagnosis and management of latent including amoxicillin and cephalexin, tend to have
tuberculosis: http://www.cdc.gov/tb/default.htm. inferior eicacy and more adverse efects compared to
the recommended agents. hey are therefore consid-
6. ANSWER: A. Campylobacter ered second-line agents for uncomplicated cystitis and
Campylobacter infection can result in Guillain-Barré are recommended when other agents cannot be used.
syndrome (GBS) several weeks after diarrhea. Approx- Gupta K, Hooton TM, Naber KG, et al. International clinical
imately 1/1000 reported Campylobacter illnesses leads practice guidelines for the treatment of acute uncomplicated
cystitis and pyelonephritis in women: A 2010 update by the
to GBS, and up to 40% of GBS in the United States
Infectious Diseases Society of America and the European
may be triggered by Campylobacter. Acute Campylo- Society for Microbiology and Infectious Diseases. Clin
bacter gastrointestinal illness can present with diarrhea Infect Dis. 2011;52:e103–e120; Food and Drug Adminis-
(possibly bloody), cramping, abdominal pain, and tration Safety Announcement; July 26, 2016; http://www.
fever, sometimes with nausea and vomiting, and can fda.gov/Drugs/DrugSafety/ucm511530.htm.
last from 2 to 10 days. It is not usually spread from one
person to another, but most cases are associated with 8. ANSWER: E. Reactivation of hepatitis B infection
contact with raw or undercooked poultry. As few as Rituximab, a monoclonal antibody to CD-20 (a
100 organisms can cause illness. Illness can also result B-lymphocyte marker), is a well-described cause of hep-
from contact with stool of an ill pet dog or cat. Antibi- atitis B reactivation characterized by redevelopment of
otics are indicated in severe cases. high levels of circulating hepatitis B virus, transaminitis,
Hughes RAC, Cornblath DR. Guillain-Barré syndrome. Lan- and in some cases a symptomatic hepatitis lare. Inactive
cet. 2005;366:1653–1666; Vucic S, Kiernan MC, Corn-
carriers of hepatitis B (surface antigen positive, but with
blath DR. Guillain-Barré syndrome: an update. J Clin
Neurosci. 2009;16:733–741.
normal liver function tests [LFTs] and low amounts of
14 C HA P T E R 1 Infectious Diseases

circulating virus) and also patients with serologic evi- and the tick was attached for long enough to trans-
dence of previous hepatitis B infection (hepatitis B core mit the infection (at least 36 hours) are at substantial
IgG positive, surface antigen negative) are both at risk risk for developing Lyme disease. he IDSA recom-
for reactivation after therapy with rituximab. Reacti- mends either of two potential management strategies
vation tends to occur early (weeks to months) after in this case: a single oral dose of doxycycline 200 mg ×
rituximab therapy in “inactive carriers,” whereas it is 1 within 72 hours of tick removal or close observation
more often a late complication of rituximab therapy in for rash or other symptoms of early Lyme disease. A
patients with serologic evidence of previous infection, full treatment course for Lyme disease (Options B and
occurring up to a year after rituximab therapy ends. D) or a single dose of antibiotics that might be active
here is very low risk of developing hepatitis C against Lyme other than the doxycycline regimen
infection as a consequence of blood transfusion in above (Option A) are not recommended. Serologies
the United States (<1 in 2 million transfusions), and are unlikely to be helpful in a case of recent exposure as
hepatitis C infection does not frequently cause an seroconversion within 1–2 days or exposure is unlikely
acute illness as described here (Option A). Transami- (Option C).
nitis would be an unusual manifestation of relapse of Wormser GP, Dattwyler RJ, Shapiro ED, et al. he clinical
lymphoma (Option B). Hepatitis A can cause an acute assessment, treatment, and prevention of Lyme disease,
symptomatic hepatitis similar to this case presenta- human granulocytic anaplasmosis, and babesiosis: clinical
tion, but this patient was immune to hepatitis A based practice guidelines by the Infectious Diseases Society of
America. Clin Infect Dis. 2006;43:1089–1134.
on prechemotherapy serologies so it is unlikely here
(Option C). Isoniazid can cause idiosyncratic drug-
induced liver injury characterized by signiicantly 11. ANSWER: C. Trimethoprim-sulfamethoxazole
elevated transaminases, but this would be an unlikely his patient has a purulent skin and soft tissue
cause of new elevation in LFTs many months after the infection that was appropriately drained and cul-
treatment course ended (Option D). tured, which is the mainstay of treatment. Uncom-
Perrillo RP, Gish R, Falck-Ytter YT. American Gastroenterological plicated purulent skin and soft tissue infections
Association Institute technical review on prevention and treat- are most often caused by Staphylococcus aureus.
ment of hepatitis B virus reactivation during immunosuppres- Although in the absence of systemic illness this
sive drug therapy. Gastroenterology. 2015;148:221–244. type of infection can be managed with drainage
alone, a recent study comparing drainage and tri-
9. ANSWER: C. Start empiric azithromycin for a 5-day methoprim-sulfamethoxazole to drainage and pla-
course. cebo found high clinical cure rates in those treated
he clinical incubation of Bordetella pertussis is with trimethoprim-sulfamethoxazole. Antibiotics
5–21 days, and patients are considered infectious active against S. aureus, MRSA in particular, are
until 3 weeks after symptom onset. B. pertussis is a recommended while cultures of purulent mate-
highly communicable disease, and patients with sus- rial are pending. Appropriate antibiotics include
pected infection should be treated pending test results. trimethoprim-sulfamethoxazole or doxycycline,
All nonpregnant patients who present with 3 weeks both of which are usually active against methicillin-
or less of symptoms (6 weeks or less if pregnant) are sensitive and resistant strains of S. aureus. Some of
potentially contagious and so should be treated. he the other antibiotic options in this question have
treatment of choice for adults is azithromycin, clar- limited activity against S. aureus (levoloxacin, clar-
ithromycin, or erythromycin. he CDC recommends ithromycin), but none of the other options are rec-
trimethoprim-sulfamethoxazole as the second-line ommended for the treatment of purulent skin and
agent to treat those with macrolide antibiotic allergies soft tissue infection. Penicillin may be active against
or intolerance. Fluoroquinolones have not been shown susceptible S. aureus infection, but this susceptibil-
to be efective as therapy. Neither vaccination nor ity pattern is rare; penicillin is more appropriate to
administration of immunoglobulin would adequately treat uncomplicated nonpurulent cellulitis, which
treat B. pertussis infection. is more often due to streptococci. Notably, rifampin
Tiwari T, Murphy TV, Moran J, et al. Recommended antimi-
is an oral antibiotic with good oral bioavailability
crobial agents for the treatment and postexposure prophy-
laxis of pertussis: 2005 CDC Guidelines. MMWR Recomm
and activity against S. aureus. However, rifampin
Rep. 2005;54(RR14):1–16. should not be used as monotherapy in the treatment
of these infections due to a low barrier to resistance.
Talan DA, Mower WR, Krishnadasan A, et al. Trimethoprim-
10. ANSWER: E. Doxycycline 200 mg by mouth × 1
sulfamethoxazole versus placebo for uncomplicated skin
In the mid-Atlantic and Northeast, including Dela- abscess. N Engl J Med. 2016;374:823–832; Stevens DL,
ware, the deer tick Ixodes scapularis is the vector for Bisno AL, Chambers HF, et al. Practice guidelines for the
Borrelia burgdorferi, which causes Lyme disease. Indi- diagnosis and management of skin and soft tissue infections:
viduals bitten by ticks in an endemic area where there 2014 update by the Infectious Diseases Society of America.
is clear evidence that the bite was due to a deer tick Clin Infect Dis. 2014;59:147–159.
CHAPTER 1 Infectious Diseases 15

12. ANSWER: E. Oral amoxicillin-clavulanate (caspofungin, micafungin, anidulafungin) is the irst-


Splenectomy results in increased vulnerability line agent for treating candidemia in an ill patient such
to overwhelming infection and sepsis due to certain as this. Fluconazole may also be a reasonable agent to
organisms, including Streptococcus pneumoniae, Hae- use initially in select patients (such as the elderly or
mophilus inluenzae, Neisseria meningitidis, and Cap- diabetic patients) who are hemodynamically stable,
nocytophaga canimorsus. C. canimorsus is an anaerobic have not previously been treated with an antifungal in
gram-negative rod that is part of canine and feline oral the azole class, and are not at risk for resistant fungi. In
lora. When inoculated into humans, C. canimorsus addition, luconazole is renally cleared and therefore
can cause severe infection in certain hosts, includ- the dose ofered in Option A would not be appropri-
ing those without a spleen. Given this risk, typically ate for a patient on hemodialysis. Isavuconazonium
asplenic patients who sustain a dog or cat bite in which (Option C) and liposomal amphotericin (Option E)
the skin is broken are treated with a short course of also likely have activity against Candida, but neither
preventative oral antibiotics. First-generation cepha- are recommended as irst-line agents for this indica-
losporins (such as cephalexin) and trimethoprim- tion. In most cases, central venous catheters need to
sulfamethoxazole are not typically active; beta-lactam/ be removed in the setting of candidemia (Option B),
beta-lactamase inhibitors and third-generation ceph- but candidemia needs to be treated as quickly as pos-
alosporins are usually active. In this case the patient sible—fast treatment can portend better outcomes, so
is well clinically so has no indication for intravenous Option B is incorrect.
antibiotics (ceftriaxone); thus amoxicillin-clavulanate Pappas PG, Kaufman CA, Andes DR, et al. Clinical practice
is the most appropriate choice. Rabies is unlikely in guideline for the management of candidiasis: 2016 update
this case because the puppy is a domestic pet. by the Infectious Diseases Society of America. Clin Infect
Butler T. Capnocytophaga canimorsus: an emerging cause of sep- Dis. 2016;62:e1–50.
sis, meningitis, and post-splenectomy infection after dog
bites. Eur J Clin Microbiol Infect Dis. 2015;34:1271–1280. 15. ANSWER: A. Bartonella henselae
All of the pathogens listed including Bartonella
13. ANSWER: C. Use of a chlorine-containing cleaning henselae, Toxocara cati, Toxoplasmosis gondii, and Yer-
agent to address environmental contamination sinia pestis can be transmitted to humans by cats. he
Standard environmental cleaning detergents are not most likely cause of this patient’s symptoms is B. hense-
sporicidal. To adequately clean the room of a patient with lae. Systemic bartonellosis can sometimes present with
Clostridium diicile infection, it is necessary to use a chlo- a primary lesion that develops 3–10 days following
rine-containing cleaning agent (1000–5000 ppm avail- inoculation from a bite or scratch from an infected cat
able chlorine) or other sporicidal agent. Even in outbreak (usually kitten). Tender lymphadenopathy may develop
settings, routine identiication of asymptomatic carriers is proximally after 1–10 weeks with overlying erythema
not recommended and treatment is not efective in reduc- that can suppurate and may last weeks. Rare complica-
ing horizontal transmission. Strict adherence to contact tions include neuroretinitis (stellate macular exudates;
precautions, including gowning and gloving on entry to “macular star”), encephalopathy/transverse myelitis, or
a patient’s room and hand hygiene with soap and water, endocarditis. In immune-suppressed individuals, dis-
are key measures in reducing horizontal transmission of seminated illness can present as bacillary angiomatosis.
C. diicile. he spore form of C. diicile is resistant to T. cati is a helminthic infection that can be passed from
killing by alcohol. Contact precautions should continue cats to humans via fecal oral transmission; it is typically
at least for the duration of the diarrhea, with many hos- asymptomatic but can cause cutaneous larva migrans.
pitals continuing contact precautions for the duration of Toxoplasmosis is a protozoan infection that similarly
the inpatient admission. here is no evidence at this time can be spread from cats to humans via fecal oral trans-
to support the use of prophylactic antibiotics to prevent mission. his infection is also often asymptomatic in
horizontal transmission. normal hosts but can cause a mono-like illness with
Cohen SH, Gerding DN, Johnson S, et al. Clinical practice cervical adenopathy and occasionally can cause retinal
guidelines for Clostridium diicile infection in adults: 2010 disease in normal hosts. Y. pestis, the cause of plague, is
update by the Society for Healthcare Epidemiology of Amer- transmitted uncommonly by feline leas and is generally
ica (SHEA) and the Infectious Diseases Society of America a rare illness in the United States.
(IDSA). Infect Control Hosp Epidemiol. 2010;31:431–455. Goldstein EJC, Abrahamian FM. Diseases transmitted by cats.
Microbiol Spectr. 2015;3:IOL5-0013-2015.
14. ANSWER: D. Start intravenous caspofungin.
he most likely cause of yeast in blood cultures 16. ANSWER: D. Country of birth
in this patient with recent antibiotic exposure and he US Preventive Services Task Force updated rec-
an indwelling hemodialysis catheter is Candida spe- ommendations for hepatitis B screening in 2014 based
cies. Based on IDSA guidelines for the management on extensive review of available data. Individuals who
of Candida infections, an echinocandin antifungal should be screened for hepatitis B infection include
16 C HA P T E R 1 Infectious Diseases

those born in countries where the prevalence of HBV intranasal eschar. he most common infection syn-
infection is higher than 2%, those born in the United dromes in this clinical context include mucormyco-
States to parents born in countries or regions where sis and aspergillosis. Appropriate empiric antifungal
the prevalence of hepatitis B infection is higher than therapy in suspected cases of fungal sinusitis includes
8% and were not vaccinated in infancy, those who are an antifungal agent active against Mucorales and
HIV-infected, injection drug users, men who have Aspergillus species such as amphotericin B or a lipid
sex with men, and household contacts or sexual part- formulation of amphotericin (which may be less neph-
ners of individuals with chronic hepatitis B infection. rotoxic in this individual with chronic kidney disease).
Based on these recommendations, the reason that this Voriconazole and caspofungin are active against Asper-
patient needs to be screened for hepatitis B infection gillus species but do not have activity against Mucora-
is her country of birth. he estimated prevalence of les when given alone. Fluconazole has no activity in
hepatitis B infection in Vietnam and Asia in general Aspergillus species or Mucorales. Two azole antifungal
is above 2%. Other regions of high hepatitis B infec- drugs, posaconazole and isavuconazole, have in vitro
tion prevalence include Africa and parts of the Middle activity against Mucorales, though typically a lipid for-
East and Central and South America. In addition to mulation of amphotericin in combination with surgi-
the above screening recommendations, the CDC also cal debridement is the treatment of choice for initial
recommends hepatitis B screening for all individuals management of these types of infections.
in whom immunosuppressive therapy (including che- Chitasombat MN, Kontoyiannis DP. Treatment of mucormy-
motherapy) is planned, individuals on hemodialysis, cosis in transplant patients: role of surgery and of old and
blood and body luid donors, all pregnant women, new antifungal agents. Curr Opin Infect Dis. 2016;29:340–
and infants of HBV-infected mothers. 345.
LeFevre ML. Screening for hepatitis B virus infection in non-
pregnant adolescents and adults: U.S. Preventive Services 19. ANSWER: E. Continue levoloxacin and observe.
Task Force recommendation statement. Ann Intern Med. his patient has Legionella pneumonia based on a
2014;161:58–66; Weinbaum CM, Williams I, Mast EE, clinical syndrome consistent with community-acquired
et al. Recommendations for identiication and public health pneumonia and a positive urinary Legionella antigen,
management of persons with chronic hepatitis B virus infec- which has high speciicity for the diagnosis of Legionella
tion. MMWR Recomm Rep. 2008;57:RR-8. serotype 1. herefore treatment of Legionella with either
levoloxacin or azithromycin, which are considered
17. ANSWER: B. Initiate systemic corticosteroids to con- irst-line therapy for Legionella pneumonia, is indicated.
trol symptoms of paradoxical tuberculosis immune Gram stain and culture of the expectorated sputum in
reconstitution inlammatory syndrome (IRIS). this case suggest that the specimen was orally contami-
he clinical syndrome is classic for paradoxical tuber- nated and does not represent the lower respiratory tract.
culosis IRIS, which is characterized by new or worsen- he growth of Candida species from sputum typi-
ing indings of tuberculosis disease within 2–3 months cally indicates colonization and does not require anti-
of initiation of antiretroviral therapy. Manifestations can fungal treatment as described in the IDSA guidelines
be seen within lung parenchyma, the central nervous sys- for the management of Candida infections (thus
tem, and at serosal surfaces (pleural efusions, ascites, or Options A–D are incorrect). his patient may have
pericardial efusions). Mild symptoms can be controlled been orally colonized with Candida due to the use of
with nonsteroidal antiinlammatory medications, but inhaled luticasone (particularly if he is not rinsing his
moderate to severe symptoms often require systemic cor- mouth after use).
ticosteroids. Multidrug-resistant tuberculosis is unlikely Phin N, Parry-Ford F, Harrison T, et al. Epidemiology and clin-
given the negative drug susceptibility test. he rapid ical management of Legionnaires’ disease. Lancet Infect Dis.
onset of symptoms is not consistent with loss of control 2014;14:1011–1021; Pappas PG, Kaufman CA, Andes
of tuberculosis infection due to poor adherence. Pneumo- DR, et al. Clinical practice guideline for the management of
cystis jirovecii is in the diferential diagnosis but less likely candidiasis: 2016 update by the Infectious Diseases Society
given the CD4 T cell count above 200 cells/µL3. of America. Clin Infect Dis. 2016;62:e1–50.
Panel on Antiretroviral Guidelines for Adults and Adolescents.
Guidelines for the use of antiretroviral agents in HIV-1-in- 20. ANSWER: A. Vancomycin + cefepime
fected adults and adolescents. Department of Health and his patient most likely has bacterial endocardi-
Human Services. <http://aidsinfo.nih.gov/contentiles/lvgu tis. Because this patient is febrile and ill appearing,
idelines/AdultandAdolescentGL.pdf/>; accessed 08.05.16. it is appropriate to start empiric antibiotics as soon as
blood cultures have been drawn in order to prevent
18. ANSWER: B. Lipid formulation of amphotericin B further complications of infection. he most common
he case patient’s presentation is consistent with pathogens in this type of presentation in an intravenous
a severe sinus infection with an angioinvasive mold drug user include Staphylococcus aureus, coagulase-neg-
resulting in a rapidly progressive infection with an ative staphylococci, viridans streptococci (β-hemolytic
CHAPTER 1 Infectious Diseases 17

streptococci, oral lora), and occasionally aerobic gram- Likewise, intravenous hydrocortisone is not recom-
negative bacilli or fungi. Empiric therapy should target mended if hemodynamic stability is restored with
these bacteria. Enterococci are a less frequent cause of luid resuscitation and vasopressors.
endocarditis, though still important to consider— Singer M, Deutschman CS, Seymour CW, et al. he hird
empiric treatment for staphylococci and streptococci International Consensus Deinitions for Sepsis and Septic
will also include empiric treatment for enterococci. Can- Shock (Sepsis-3). JAMA. 2016;315:801–810; Dellinger
dida endocarditis occurs in injection drug users but is RP, Levy MM, Rhodes A, et al. Surviving sepsis campaign:
international guidelines for management of severe sepsis
less common than the bacterial pathogens.
and septic shock: 2012. Crit Care Med. 2013;41:580–637.
he best empiric therapy among the choices given
is vancomycin and cefepime, which would treat S.
aureus (including MRSA), enterococci, streptococci, 22. ANSWER: C. Caspofungin
and aerobic gram-negative organisms (including Pseu- Neutropenic patients with fever that persists or
domonas). Another reasonable regimen would include recurs after 4–7 days of empiric antibiotics are at high
vancomycin and ceftriaxone. Although Option B, risk for fungal infection, particularly due to Candida
which includes vancomycin, gentamicin, and rifampin, and Aspergillus species. Neutropenic patients with this
would have activity against the organisms of concern, fever pattern should be treated with empiric antifun-
this is the regimen recommended for treatment of gal therapy. Based on IDSA guidelines, appropriate
prosthetic valve MRSA endocarditis. Rifampin has a antifungal therapy at this juncture would include an
low barrier of resistance to bacteria so is not typically echinocandin (such as caspofungin or micafungin),
used during active bacteremia and is typically only voriconazole, or a preparation of amphotericin.
Freifeld AG, Bow EJ, Sepkowitz KA, et al. Clinical practice
indicated to treat staphylococcal prosthetic valve endo-
guideline for the use of antimicrobial agents in neutropenic
carditis or speciic rare causes of endocarditis such as patients with cancer: 2010 update by the Infectious Diseases
Brucella. he remaining antibiotic treatment options Society of America. Clin Infect Dis. 2011;52:e56–e93.
do not adequately target the most common pathogens
as listed above.
Baddour LM, Wilson WR, Bayer AS, et al. Infective endocardi-
23. ANSWER: E. Blood smear, EBV serologies, CMV
tis in adults: diagnosis, antimicrobial therapy, and manage- serologies, HIV 1/2 antigen/antibody, HIV RNA
ment of complications: a scientiic statement for healthcare his patient has a “mono-like illness,” characterized
professionals from the American Heart Association. Circu- by fever, sore throat, malaise, fatigue, rash, and lymph-
lation. 2015;132:1435–1486. adenopathy. Mild hepatitis and pancytopenia may also
be a part of the syndrome. he most common causes
21. ANSWER: C. Administration of 30 mL/kg of crys- of this syndrome in young adults are viral pathogens,
talloid including EBV and CMV for those who were not
his patient is at risk for sepsis based on the clinical already infected during childhood. Serologies for both
criteria of altered mental status, increased respiratory EBV and CMV are helpful diagnostics here. Acute
rate (>22 breaths per minute), and low systolic blood retroviral syndrome, or acute HIV, may also present
pressure (<100 mm Hg), which make up the quick as a mono-like illness, often with concurrent diarrhea
Sequential Organ Failure Assessment (qSOFA). he and weight loss just as this patient had. he incidence
qSOFA and the more detailed SOFA score, which is of new HIV infections in the United States is high-
based on Pao2, platelet count, bilirubin, mean arterial est among men who have sex with men at present,
pressure, Glasgow coma scale, serum creatinine, and so this patient has a documented risk factor as well.
urine output, allow clinicians to identify patients with Acute HIV is the most likely diagnosis for this patient.
sepsis who are at increased risk for mortality to allow In acute HIV, the HIV antibody is often still nega-
for appropriate early intervention. tive, before seroconversion, but the HIV p24 antigen
he Surviving Sepsis Campaign recommends that is positive. Fourth-generation HIV tests check serum
the following be completed within 3 hours: mea- for HIV antibodies and p24 antigen. he CDC rec-
surement of lactate, obtaining blood cultures before ommends the following testing algorithm for patients
administration of antibiotics, administration of broad- with suspected established or acute HIV infection:
spectrum antibiotics, and administration of 30 mL/kg initial HIV 1/2 antigen/antibody testing; if the test is
of crystalloid for hypotension or lactate ≥4 mmol/L. reactive, then the sample is relexively tested specii-
here is good evidence that early antibiotics (within cally for HIV1 and HIV2 antibodies. his antibody
1 hour of recognizing sepsis) improves patient out- diferentiation test would be positive for either HIV1
comes. Norepinephrine is the irst-choice vasopressor or HIV2 antibodies in established infection, whereas
to maintain a mean arterial pressure ≥65; however, it may be negative in acute infection. hus patients
vasopressors should be reserved for hypotension that is with discordant results on the HIV 1/2 antigen/anti-
nonresponsive to luid resuscitation. Dopamine is not body and antibody diferentiation test (positive/nega-
recommended except in highly select circumstances. tive) should have further testing with an HIV RNA
18 C HA P T E R 1 Infectious Diseases

(or viral load), which should be positive and is usually high risk for mortality, two drugs that treat Pseudomo-
very high in acute HIV. In cases where suspicion is nas, including beta-lactams, quinolones, or aminogly-
high for acute HIV, like this case, it is also reasonable cosides (but preferably not to include two agents in
to check HIV RNA upfront because it becomes posi- the same class), and a single agent to treat for S. aureus,
tive in acute infection a few days before the HIV anti- including MRSA (either vancomycin or linezolid), are
gen/antibody test. recommended. herefore among the choices given,
Although hepatitis serologies may be helpful at some cefepime, levoloxacin, and vancomycin would be the
point if the liver function tests continue to be abnor- best choice. Current guidelines emphasize the impor-
mal, viral hepatitis would not explain the full constel- tance of referring to local antibiograms data to choose
lation of this patient’s symptoms or lab abnormalities, the best antibiotic combination for patients at each
and the transaminases are not high enough to be con- center. Antibiotics can be adjusted based on clinical
sistent with acute viral hepatitis, so these would not improvement and culture results at 48 to 72 hours.
be the appropriate next tests to order. It is appropriate Kalil AC, Metersky ML, Klompas M, et al. Management of
to consider the possibility of new sexually transmitted adults with hospital-acquired and ventilator-associated
infections (STIs) given his new sexual partners, and pneumonia: 2016 clinical practice guidelines by the Infec-
secondary syphilis can also present with fever and rash tious Diseases Society of America and the American ho-
racic Society. Clin Infect Dis. 2016;63:e61–e111.
and lymphadenopathy but would be unlikely to present
with diarrhea, weight loss, and pancytopenia. An echo-
cardiogram might be an appropriate test if the blood 25. ANSWER: A. Clotrimazole troches
cultures are positive, but there are no other clues spe- he patient in this case has clinical evidence of oral
ciic to endocarditis here aside from the fever. A blood candidiasis (thrush). He recently received treatment
smear and LDH may be appropriate tests at this point with systemic steroids and antibiotics and is also on a
given the patient’s pancytopenia of uncertain etiology, chronic inhaled steroid, all of which raise his potential
but a bone marrow biopsy should not be pursued until risk for thrush. In order to minimize the possibility
further noninvasive diagnostics have been performed. that the thrush resulted from the use of an inhaled ste-
Centers for Disease Control and Prevention and Association of roid, reviewing proper oral hygiene after use could be
Public Health Laboratories. Laboratory testing for the diag- helpful. Despite these risk factors, it is also reasonable
nosis of HIV infection: updated recommendations. Avail- to consider HIV testing; the CDC currently recom-
able at <http://stacks.cdc.gov/view/cdc/23447/>. Published mends testing for all patients in the health care set-
June 27, 2014. Accessed 08.05.16. ting. Appropriate treatment for mild thrush includes
clotrimazole troches or miconazole buccal tablets; oral
24. ANSWER: E. Cefepime + levoloxacin + vancomycin nystatin solution or pastilles is also an alternative. Sys-
his patient has hospital-acquired pneumonia (HAP), temic oral luconazole is typically reserved for moder-
which is deined as “pneumonia not incubating at the ate or severe oral thrush. Posaconazole and micafungin
time of hospital admission and occurring 48 hours or are broader spectrum systemic antifungal agents than
more after admission” by the IDSA. he appropriate luconazole and are not indicated for the treatment of
antibiotic choice in this setting depends on three patient mild oral thrush in a normal host, though posacon-
factors: azole is sometimes used for luconazole-refractory
1. Risk for infection with MRSA thrush in a compromised host. Chlorhexidine mouth-
• Risk for MRSA is increased if the patient has wash is approved for treatment of gingivitis but does
received IV antibiotic therapy in the last 90 not have antifungal activity and will not treat oral
days or is hospitalized in a unit where more thrush.
than 20% of S. aureus isolates are methicillin- Pappas PG, Kaufman CA, Andes DR, et al. Clinical practice
resistant or if this statistic is not known. guideline for the management of candidiasis: 2016 update
2. Risk for mortality by the Infectious Diseases Society of America. Clin Infect
• Risk for mortality is increased if the patient Dis. 2016;62:e1–50.
requires ventilatory support for or has
septic shock due to HAP. 26. ANSWER: B. Colistimethate
3. Risk for infections due to multidrug-resistant org- Acinetobacter baumannii is an aerobic gram-negative
anisms including Pseudomonas species. coccobacillus that can cause hospital outbreaks with
• Risk for resistant organisms is increased high rates of mortality. Outbreaks are typically asso-
if the patient has received IV antibiotic ciated with colonized respiratory support equipment,
therapy in the last 90 days. irrigation solutions, and intravenous solutions. he
Based on the patient’s previous course of IV antibi- most common infections include ventilator-associated
otics for bacteremia within the last 3 months, he is at pneumonia, infection of surgical wounds and burns,
increased risk for both multidrug-resistant pathogens and bacteremia. Over 50% of A. baumannii isolates
and MRSA. For patients in this setting or in those at in US hospitals are multidrug resistant. Colistimethate
CHAPTER 1 Infectious Diseases 19

(colistin) is virtually always active in vitro. he pres- eicacy, of these agents. Efavirenz is both a substrate
ence of a beta-lactamase means that both cefepime and inducer of isoforms of the cytochrome P450 sys-
and piperacillin/tazobactam, the two most active beta- tem, but interactions with efavirenz are generally less
lactam antibiotics against A. baumannii, are likely to common and less severe, though the interaction with
be inefective. he presence of a carbapenemase means methadone is unpredictable and can be problem-
that imipenem/cilastatin, meropenem, and doripenem atic. Inhaled albuterol, ophthalmic antihistamines,
are all unlikely to be efective. Finally, most multidrug- oral cetirizine, and oral montelukast would not be
resistant A. baumannii isolates are resistant to luoro- expected to cause a problematic drug interaction for
quinolones. Even if the isolate tests susceptible to a this patient.
luoroquinolone, resistance often develops during use. Panel on Antiretroviral Guidelines for Adults and Adolescents.
Fishbain J, Peleg AY. Treatment of Acinetobacter infections. Clin Guidelines for the use of antiretroviral agents in HIV-
Infect Dis. 2010;51:79–84. 1-infected adults and adolescents. Department of Health
and Human Services. Available at http://www.aidsinfo.nih.
27. ANSWER: D. Inhaled luticasone gov/ContentFiles/AdultandAdolescentGL.pdf. Accessed
08.08.16.
here are many possible problematic drug–drug
interactions with antiretroviral medications and com-
monly prescribed drugs. Physicians should review 28. ANSWER: D. Bacteroides fragilis
medication lists for any possible drug–drug interac- Bacteroides fragilis is an anaerobic gram-negative
tions before prescribing new medications to patients bacteria that colonizes the intestines and is cultured
who are currently taking antiretroviral medications. from the blood in the setting of bowel perforation.
Computer software applications that predict drug It is not a common cause of endocarditis. Common
interactions from a physician-entered medication list pathogens that cause endocarditis include Staphylo-
are particularly helpful in this setting. he most prob- coccus aureus, the viridans streptococci, Streptococcus
lematic antiretroviral agent in terms of risk of drug gallolyticus (previously known as Streptococcus bovis),
interactions is ritonavir. Ritonavir is the most potent the HACEK organisms (Haemophilus parainluenzae,
cytochrome P450 3A4 inhibitor of all currently avail- Haemophilus aphrophilus, Haemophilus paraphrophilus,
able medications. he other protease inhibitors, such Actinobacillus, Cardiobacterium hominis, Eikenella cor-
as atazanavir and darunavir, and the pharmacologic rodens, Kingella kingae), and enterococci (community
booster agent cobicistat (available in several cofor- acquired, without another obvious source). Positive
mulations and alone), also inhibit CYP 3A4. hese blood cultures from at least two separate cultures with
medications interact with many commonly prescribed any of those organisms are considered major criteria
drugs, including warfarin, the statins, combination for the diagnosis of endocarditis by the modiied Duke
estrogen/progesterone oral contraceptives, clarithro- criteria, or highly positive antibody titer to Coxiella
mycin, rifampin, amiodarone, benzodiazepines, carba- burnetii (because this organism is exceedingly diicult
mazepine, phenytoin, sildenail, tadalail, meperidine, to culture). he other major criterion for diagnosis of
methadone, and peripherally administered cortico- endocarditis is an echocardiogram with indings con-
steroids. Peripherally administered corticosteroids sistent with infective endocarditis (mobile intracardiac
(inhaled luticasone, in this case) may reach systemic mass, abscess, new dehiscence of prosthetic valve, or
levels in patients who are taking ritonavir, which new valvular regurgitation). A diagnosis of endocar-
can lead to corticosteroid excess (features of Cush- ditis by Duke criteria requires fulillment of either
ing syndrome) and then later lead to corticosteroid two major criteria (as listed previously), or one major
insuiciency when the medication is withdrawn. he and three minor criteria, or ive minor criteria. his
systemic efects of peripherally administered cortico- patient meets two minor criteria for the diagnosis of
steroids with ritonavir are unpredictable and thus are endocarditis at the time of presentation: fever and
diicult to monitor and treat. Peripherally adminis- predisposing cardiac lesion (the bicuspid valve with
tered corticosteroids, including inhaled, intranasal, regurgitation). Other minor criteria are as follows:
injected (joint injections for treatment of pain), and other predisposing risk (injection drug use), evidence
even ophthalmic, should be avoided in patients tak- of emboli (arterial emboli, pulmonary infarcts, Jane-
ing ritonavir. If these medications are necessary, then way lesions, conjunctival hemorrhage), immunologic
an HIV specialist should be consulted. In addition complications (glomerulonephritis, Osler nodes), or
to the aforementioned list of drug–drug interactions, positive blood cultures that do not meet the major
there are other common drug–drug interactions with criteria or serologic evidence of an infection with an
antiretroviral medications. Both atazanavir and rilpi- organism consistent with infective endocarditis not
virine require stomach acid for absorption to thera- satisfying a major criterion (e.g., Bartonella henselae).
peutic levels, so coadministration of acid blockers (H2 he diagnosis of infective endocarditis is “possible” if
blockers and especially proton pump inhibitors) can only one major and one minor criteria are fulilled, or
lead to decreased blood levels, and therefore decreased if three minor criteria are fulilled.
20 C HA P T E R 1 Infectious Diseases

Baddour LM, Wilson WR, Bayer AS, et al. Infective endocardi- as opportunistic infections and AIDS-related malig-
tis in adults: diagnosis, antimicrobial therapy, and manage- nancies (e.g., Kaposi sarcoma), there are other risks
ment of complications: a scientiic statement for healthcare with prolonged HIV infection. Prolonged HIV infec-
professionals from the American Heart Association. Circu- tion is associated with increased risk of cardiovascu-
lation. 2015;132:1435–1486.
lar disease and non–AIDS-related malignancies, such
as lung cancer. Health care maintenance, including
29. ANSWER: B. A few days after initiation of clar- addressing modiiable risk factors and pursuing age-
ithromycin and ethambutol, but within 2 weeks appropriate cancer screening, is extremely important
Antiretroviral therapy (ART) should be started in these patients. For this patient the most important
within 2 weeks of diagnosis of most opportunistic modiiable risk factor for cardiovascular disease and
infections, with the exception of cryptococcal men- malignancy is his smoking. Among all of the health
ingitis and tuberculosis meningitis. here is a risk care maintenance items that could be addressed with
of immune reconstitution inlammatory syndrome the patient during this visit, smoking cessation (if
(IRIS) in patients who start ART with low CD4 this can be achieved) is the most likely to lead to a
count, high viral load, and active opportunistic infec- reduction in morbidity and mortality. It is important
tion. IRIS may be less pronounced if the opportunistic to address smoking cessation with him through coun-
infection is treated irst. However, IRIS may be treated seling (use of motivational interviewing is especially
with corticosteroids if needed. Patients have increased helpful here) and discussion of nicotine replacement
mortality from AIDS-related causes if ART is not therapy (nicotine patches, etc.) and other medications
started early enough. It may be appropriate to start to aid in smoking cessation (varenicline, bupropion).
treatment for opportunistic infections irst to ensure his patient does not meet criteria to start prophylaxis
tolerance of those medications for a couple of days for Pneumocystis pneumonia because his CD4 count is
prior to ART and start control of those infections, but well above 200, and he is not at risk for reactivation of
then ART should be started soon afterwards, within latent toxoplasmosis because his CD4 count is greater
2 weeks of the diagnosis of opportunistic infection. than 100. A discussion about alcohol use to screen for
here is no reason to wait until hospital discharge to warning signs of abuse or dependence may be appro-
start ART. ART may be started during hospitalization priate at this visit, but his current reported number of
and transitioned to the outpatient setting in the same alcoholic beverages per week is below the threshold of
way that other new medications (e.g., statins, beta- concern for men, so this is less important to address
blockers, and clopidogrel after new cardiac event) are than smoking cessation. he patient should be referred
started during hospitalization and transitioned to the for colonoscopy screening at age 50, but in the absence
outpatient setting. ART should not be started in a of symptoms or a personal or family history of polyps
staggered fashion, one at a time over weeks. or colon cancer, there is no indication to refer for early
Zolopa A, Andersen J, Powderly W, et al. Early antiretroviral
colonoscopy screening.
therapy reduces AIDS progression/death in individuals with
acute opportunistic infections: a multicenter randomized
strategy trial. PLOS One. 2009;4:e5575; Panel on Antiretro- 31. ANSWER: A. Vancomycin, ceftriaxone, ampicillin
viral Guidelines for Adults and Adolescents. Guidelines for his patient is an elderly man who is living in a
the use of antiretroviral agents in HIV-1-infected adults and close community at his assisted living facility, now
adolescents. Department of Health and Human Services. presenting with probable bacterial meningitis. he
Available at http://www.aidsinfo.nih.gov/ContentFiles/Adu most common pathogens in this case are Streptococ-
ltandAdolescentGL.pdf. Accessed 08.08.16. cus pneumoniae, Neisseria meningitidis, and Listeria
monocytogenes. he combination of vancomycin and
30. ANSWER: C. Counsel patient to quit smoking, and high-dose ceftriaxone is used empirically when S.
discuss medications and supports for smoking ces- pneumoniae meningitis is a possibility because of the
sation. small but signiicant prevalence of penicillin-resistant
his patient has well-controlled HIV with a CD4 and even cefotaxime-resistant S. pneumoniae. Ampi-
count well above the range that would be concerning cillin is used for treatment of Listeria meningitis.
for opportunistic infections. If he continues to take he other antibiotic options that are listed are not
his antiretrovirals regularly, then he should continue to appropriate for this patient. his patient has no his-
have a robust immune system and low risk of oppor- tory of neurosurgical procedures, so he does not need
tunistic infections. In fact, patients with HIV who are empiric treatment with an antipseudomonal cepha-
diagnosed early (before diagnosis of AIDS), started losporin, such as cefepime or ceftazidime, or a car-
on antiretroviral therapy quickly, and maintained on bapenem, such as imipenem or meropenem.
antiretroviral therapy without interruptions are esti- he CSF examination of this patient shows dei-
mated to have a life expectancy that approaches that nite predominance of neutrophils with minimal lym-
of the general population. Although this patient is phocytes and low glucose, which is more suggestive
unlikely to develop AIDS-related complications, such of bacterial meningitis rather than viral meningitis,
CHAPTER 1 Infectious Diseases 21

so acyclovir is not required for initial treatment. never indicated as a part of treatment of an acute ill-
he combination of ampicillin and gentamicin may ness, so Option E is incorrect. his patient should
be useful for such pathogens as ampicillin-sensitive be treated with supportive care, including symptom
enterococci or Streptococcus agalactiae (group B Strep- management—all of the medications in Option C
tococcus), but it does not include appropriate empiric may be used for symptom management, so this is the
treatment for S. pneumoniae or N. meningitidis. Fun- correct answer.
gal meningitis is rare, seen primarily in patients who Chow AW, Benninger MS, Brook I, et al. IDSA clinical practice
are immunocompromised or those who have been guideline for acute bacterial rhinosinusitis in children and
instrumented or undergone epidural injections with adults. Clin Infect Dis. 2012;54:e73–e112.
contaminated material such as in the 2012 outbreak
involving steroid injections contaminated with Exse- 33. ANSWER: E. Allergy consultation and admission
rohilum rostratum. his patient does not have a his- for desensitization to penicillin in order to facili-
tory of immunocompromise, instrumentation, or tate treatment with penicillin G 2.4 million units
epidural injections of medications, so fungal menin- IM once weekly for 3 weeks
gitis is extremely unlikely, and empiric therapy with Syphilis infection during pregnancy can have
amphotericin B is not appropriate. serious consequences for both the mother and
Kainer MA, Reagan DR, Nguyen DB, et al. Fungal infec- fetus if left untreated. herefore it is important to
tions associated with contaminated methylprednisolone in treat immediately with the most aggressive, data-
Tennessee. N Engl J Med. 2012;367:2194–2203; Tunkel supported efective treatment for syphilis, which is
AR, Hartman BJ, Kaplan SL, et al. Practice guidelines for penicillin. Pregnant women who are diagnosed with
the management of bacterial meningitis. Clin Infect Dis. syphilis should always be treated with penicillin,
2004;39:1267–1284. even if this requires hospitalization for desensitiza-
tion to penicillin, in the case of penicillin allergy.
32. ANSWER: C. Oral dextromethorphan, inhaled alb- All other treatments for syphilis are potentially less
uterol, and intranasal ipratropium efective, and doxycycline is contraindicated during
his patient has rhinosinusitis that is improving pregnancy. his patient may be presumed to have
without antibiotic treatment already, based on ces- late latent syphilis, as she is asymptomatic with
sation of fevers 3 days ago and only mild residual infection of unknown duration, so treatment with
symptoms. Acute rhinosinusitis (“sinusitis”) is an weekly IM penicillin for 3 weeks would be appro-
extremely common condition, especially among priate. Follow-up RPR titers should be monitored
adults age 45–74 years, and the cause is viral in more to ensure that they decrease appropriately.
than 90% of cases, with bacterial infection account- Workowski KA, Bolan GA. Sexually transmitted diseases treat-
ing for less than 10% of cases. Symptoms of upper ment guidelines, 2015. MMWR Recomm Rep. 2015;64:
respiratory tract infections or sinusitis may include 1–137.
nasal congestion, headache, ear pain, cough, fever,
and purulent nasal discharge. he indications for 34. ANSWER: D. Ceftriaxone 250 mg IM single dose +
antibiotic treatment of sinusitis are when symptoms azithromycin 1 g orally single dose
suggest bacterial infection with any one or combina- his patients has uncomplicated gonococcal urethri-
tion of three patterns: (1) persistent or not improv- tis. Fluoroquinolone-resistant strains of Neisseria gon-
ing symptoms (≥10 days); (2) severe symptoms for orrhoeae are now disseminated throughout the United
≥3–4 days (including temperature >39°C or 102°C, States. So, luoroquinolones have not been recom-
or 3–4 consecutive days of sinus pain at the begin- mended for treatment of gonorrhea in the United States
ning of the illness); or (3) worsening or “double- since 2007 (Option A). In addition, Asian and Euro-
sickening” (onset of new/worsening symptoms as pean countries have reported ceixime-resistant strains
viral upper respiratory infection is starting to resolve and treatment-failures. In the United States, gonorrhea
after 5–6 days, indicating bacterial superinfection). surveillance in the last decade has also suggested that
his patient meets none of these criteria; therefore the eicacy of ceixime may be reduced, and strains not
antibiotic treatment including moxiloxacin (Option susceptible to ceixime are also typically not susceptible
A), amoxicillin-clavulanate (Option B), and ceftriax- to tetracyclines like doxycycline. In 2015 the CDC
one (Option D) are not indicated. Furthermore, for changed their recommendations for the treatment of
patients who do meet criteria for therapy and are not uncomplicated cervical, urethral, and rectal gonorrhea
penicillin-allergic, quinolones and third-generation infection to a single preferred regimen of intramuscular
cephalosporins are no longer considered irst-line ceftriaxone and single-dose oral azithromycin. Ceftri-
therapy; amoxicillin-clavulanate remains the treat- axone, also a third-generation cephalosporin like ceix-
ment of choice for patients who do need treatment. ime, remains part of the regimen of choice because the
Inluenza vaccine, although not contraindicated in intramuscular delivery leads to long-lasting high levels
this situation because the patient is now afebrile, is of antibiotic relative to a single oral dose of ceixime.
22 C HA P T E R 1 Infectious Diseases

Ceixime remains an alternative option for treatment nodules and hemoptysis may be due to fungal infec-
of uncomplicated gonorrhea when given with a single tion or tuberculosis, or nontuberculous mycobac-
dose of azithromycin, but not in combination with terial infection. However, when the entirety of the
doxycycline (Option C). Concurrent treatment with patient’s history is reviewed, the salient points are
azithromycin in either case is important because most as follows: no obvious risk factors for tuberculosis
gonococci are also susceptible to azithromycin even or immunosuppression at baseline, chronic sinusitis
with diminished susceptibility to cephalosporins. Con- that was unresponsive to repeated treatments with
current treatment may also decrease the rates of devel- antibiotics, new pulmonary nodules and hemopty-
opment of antibiotic-resistant gonorrhea and also treats sis, and blood tests showing anemia and hematuria.
for the possibility of concomitant Chlamydia coinfec- Taken together, this syndrome is most consistent
tion in cases where this infection is present. with granulomatosis with polyangiitis (previously
Workowski KA, Bolan GA. Sexually transmitted diseases treat- known as Wegner granulomatosis). he American
ment guidelines, 2015. MMWR Recomm Rep. 2015;64:1– College of Rheumatology–endorsed diagnostic cri-
137. teria for granulomatosis with polyangiitis stipulate
that patients should meet at least 2 of 4 criteria that
35. ANSWER: B. Viral culture of a swab of the ulcers include “nasal or oral inlammation,” abnormal chest
he most likely diagnosis in this case is anogeni- imaging, abnormal urinary sediment, and granu-
tal herpes simplex virus (HSV) infection. he features lomatous inlammation of biopsy. Most patients
that are consistent with this diagnosis are the grouped with active granulomatosis will have positive serum
ulcerations, which are painful, with absence of sig- ANCA testing (up to 80%), especially if the disease
niicant lymphadenopathy or rectal discharge, which is severe, but biopsy would be deinitive. AFB smears
would be expected with some other conditions such from sputum or from biopsy samples would be posi-
as gonorrhea or lymphogranuloma venereum (LGV). tive if the patient had tuberculosis or nontubercu-
Diagnostics for active HSV infection are somewhat lous mycobacterial infection. Serum galactomannan
limited, and the diagnosis is often made clinically fol- would be positive if the patient had invasive aspergil-
lowed by somewhat empiric treatment. Viral culture losis. Serum IGRA may be positive in patients with
of a swab of the lesion has a high speciicity, but the tuberculosis.
sensitivity is limited and operator dependent. In cer- Leavitt RY, Fauci AS, Bloch DA, et al. he American College of
tain institutions the direct luorescent antibody test- Rheumatology 1990 criteria for the classiication of Wegen-
ing (DFA) of a scraping of the base of the lesion may er’s granulomatosis. Arthritis Rheum. 1990;33:1101–1107.
be highly sensitive and speciic, but it is also operator
dependent and therefore not consistent across insti- 37. ANSWER: A. Monitoring for early signs and symp-
tutions. PCR for HSV is both sensitive and speciic toms of urinary tract infection, with expedited
from CSF during episodes of HSV meningitis and early urinalysis, urine culture, and empiric treat-
can also be checked from a swab of an ulcer. Serologic ment while awaiting culture results when symp-
testing for HSV type-speciic antibodies will demon- toms develop
strate infection that was acquired at some point in the Patients with chronic indwelling urinary catheters
past if the patient is having a recurrent outbreak and often have pyuria and bacteria even in the absence of
will develop within the irst several weeks of infection other signs or symptoms of active urinary tract infec-
for new exposures, but serologies are not helpful in tion. Routine screening with urinalysis and urine cul-
diagnosing an active outbreak. Urine NAAT probes ture is not recommended in these patients because this
are helpful for diagnosis of gonorrhea or chlamydia will lead to overtreatment of asymptomatic patients,
but not helpful for diagnosis of HSV. Bacterial stud- and it increases the risk for antibiotic-resistant infec-
ies would not be expected to be positive during active tions in the future. Early diagnosis and treatment,
HSV infection, which is a viral infection. based on prior available culture results while waiting
Workowski KA, Bolan GA. Sexually transmitted diseases treat- for pending acute cultures, is the safest approach to
ment guidelines, 2015. MMWR Recomm Rep. 2015;64:1– management to decrease the risk of overtreatment.
137. Antibiotic treatments into the urinary drainage bag
are not recommended, and chronic prophylaxis with
36. ANSWER: C. Serum test for antineutrophilic cyto- methenamine salts or empiric broad antibiotics (such
plasmic antibodies (ANCA) as ciproloxacin) without any review of culture data or
his question stem describes a patient with an prior infections are not recommended.
“infection mimicker,” which can be diagnostically Hooton TM, Bradley SF, Cardenas DD, et al. Diagnosis, pre-
challenging. Many of the features of this patient’s ill- vention, and treatment of catheter-associated urinary tract
ness may be consistent with an infectious etiology— infection in adults: 2009 International Clinical Practice
chronic sinusitis, especially in immunocompromised Guidelines from the Infectious Diseases Society of America.
patients, may be due to fungal infection. Pulmonary Clin Infect Dis. 2010;50:625–663.
CHAPTER 1 Infectious Diseases 23

38. ANSWER: E. Penicillin typically spread directly from human to human in a


Recurrent cellulitis with episodes as described for teacher-student relationship, and pregnant women
this patient (sudden onset of fever, systemic symp- can keep a pet cat as long as contact with feces is
toms, and pain and erythema of the leg) is commonly minimized (e.g., another person cleans the litter box)
attributed to recurrent streptococcal infection, though and hands are washed carefully after any potential
the microbiologic diagnosis is not conirmed in most exposure.
cases. Risk factors for recurrent cellulitis may include Montoya JG, Remington JS. Management of Toxoplasma
prior surgery or prior infections in the afected limb, gondii infection during pregnancy. Clin Infect Dis.
lower-extremity edema, poor hygiene status, skin 2008;47(4):554–566; American College of Obstetricians
ulcerations, and onychomycosis. A randomized, pla- and Gynecologists. Practice bulletin no. 151: cytomega-
lovirus, parvovirus B19, varicella zoster, and toxoplasmo-
cebo-controlled trial of chronic suppressive/prophy-
sis in pregnancy. Obstet Gynecol. 2015;125:1510–1525;
lactic oral penicillin showed that this was an efective Lopez A, Dietz VJ, Wilson M, et al. Preventing congenital
strategy to decrease recurrences of cellulitis; incidence toxoplasmosis. MMWR Recomm Rep. 2000;49:59–68.
of cellulitis was 37% among placebo recipients and
22% among penicillin recipients during the follow-
up period. Many of the antibiotic options listed have 40. ANSWER: D. Vancomycin and piperacillin-tazobactam
good eicacy at treating staphylococcal infections, and his patient has poorly controlled diabetes with
some have good eicacy at treating streptococcal infec- a new progressive soft tissue infection arising from
tions (e.g., levoloxacin), but only penicillin has been a nonhealing toe ulcer. he black areas of the toe
proven to reduce risk of recurrence of cellulitis in a are concerning for necrotic tissue, which in this
placebo-controlled clinical trial. case would raise the concern for “wet gangrene,” or
homas KS, Crook AM, Nunn AJ, et al. Penicillin to prevent infected gangrenous tissue. his patient requires surgi-
recurrent leg cellulitis. N Engl J Med. 2013;368:1695–1703; cal debridement and may require amputation of the
Stevens DL, Bisno AL, Chambers HF, et al. Practice guide- toe, depending on the extent of the infection. Based
lines for the diagnosis and management of skin and soft on the presence of his local symptoms (purulent drain-
tissue infections: 2014 update by the Infectious Diseases age, tenderness, pain), the size of the ulcer (>2 cm),
Society of America. Clin Infect Dis. 2014;59:147–159. and his systemic signs and symptoms of infection
(fever >100.4°F and white blood cell count >12,000
39. ANSWER: A. Cook meat to “well done.” cells/µL), this patient has a severe infection based on
Toxoplasma gondii is a parasite found in soil and IDSA classiications. Antibiotics should be started
contaminated or undercooked foods. Cats are the immediately while awaiting results of cultures and
deinitive host and shed oocytes that eventually operative management. Cultures should be obtained
become infectious to humans or other animals if con- from deep tissue typically by biopsy or debride-
sumed (typically through poorly washed produce, ment after the wound has been cleaned, as cultures
undercooked meat, or inadequate hand washing after obtained from supericial swabs of the wound may be
soil exposure). Acute infection may be associated with less accurate. For severe infections, empirical therapy
malaise, fever, and lymphadenopathy, but it is often for Staphylococcus aureus, including MRSA, as well as
asymptomatic. Acute infection during pregnancy gram-negative bacilli, including Pseudomonas aeru-
may result in congenital infection of the newborn, ginosa when risk factors such as a high prevalence of
which can cause long-term complications such as Pseudomonas infection are present, is indicated. Of the
blindness. Early diagnosis and treatment during preg- listed antibiotic regimens, only piperacillin-tazobac-
nancy is essential to preventing complications. How- tam and vancomycin treat for these possibilities.
ever, prevention of infection is ideal. he CDC and Lipsky BA, Berendt AR, Cornia PB, et al. 2012 Infectious Dis-
the American College of Obstetrics and Gynecology eases Society of America clinical practice guideline for the
recommend that pregnant women be advised about diagnosis and treatment of diabetic foot infections. Clin
several lifestyle changes to reduce the risk of exposure. Infect Dis. 2012;54:132–173.
his includes careful washing of fruits and vegetables
before consumption because they may be contami- 41. ANSWER: B. Valacyclovir 1000 mg orally three
nated with cysts from the soil. All meats should be times daily for 14 days
cooked to “well done” because uncooked meats may his patient has Ramsay Hunt syndrome type II,
also carry infection. Although pork products may also otherwise known as herpes zoster oticus. his is a rare
carry infection, they do not need to be universally syndrome that is caused by reactivation of herpes zos-
avoided as long as they are cooked properly. Lastly, ter virus in the geniculate ganglion, which leads to a
pregnant women should wear gloves and carefully triad of symptoms including (1) ear pain, tinnitus, and
wash hands afterwards when they anticipate soil expo- vertigo, (2) ipsilateral peripheral facial nerve paraly-
sure, such as with gardening (but it need not be given sis, and (3) vesicles in the external auditory canal, ear,
up as long as gloves are worn). Toxoplasmosis is not mouth, or anterior two-thirds of the tongue. Patients
24 C HA P T E R 1 Infectious Diseases

may also experience altered sensation in the ear canal Risk for PCP in patients treated with corticosteroids
and loss of moisturization of the eyes and mouth. depends on the dose and duration of the steroids. In
he concurrence of facial droop with ear canal symp- a study of 116 patients with PCP who did not have
toms, changes in mucous membranes moisture, and HIV, 91% had been on corticosteroids when PCP
skin lesions make this syndrome unique. his zoster was diagnosed, and the mortality rate was 49%. he
reactivation is treated with acyclovir or valacyclovir. median steroid dose and duration in this study was 30
he addition of corticosteroids, such as prednisone mg of prednisone per day for 12 weeks, but it was seen
to antiviral therapy, is controversial but has not been in patients on doses as low as 16 mg per day and for
shown to be of beneit in a randomized study. he durations as short as 8 weeks.
facial droop alone can be a symptom of Lyme disease he features of the presentation that are consistent
(especially with the epidemiologic risk factors listed with Pneumocystis pneumonia include prolonged cor-
for this patient) and would be appropriately treated ticosteroid exposure (especially in the context of recent
with doxycycline but would not explain the patient’s tapering), cough and shortness of breath, which are
other symptoms. Otitis externa may be treated with progressive despite antibiotics; signiicant hypoxia on
otic ciproloxacin drops, and if concurrent otitis media presentation; and bilateral iniltrates on chest imaging.
was suspected, then additional treatment with amoxi- he serum β-d-glucan assay may be positive in the set-
cillin-clavulanate would be appropriate. ting of Pneumocystis pneumonia, especially in immu-
Worme M, Chada R, Lavallee L. An unexpected case of Ramsay nocompromised patients, and also in some other
Hunt syndrome: case report and literature review. BMC Res fungal infections. Treatment is with trimethoprim-
Notes. 2013;6:337; Kansu L, Yilmaz I. Herpes zoster oticus sulfamethoxazole, and corticosteroids should be added
(Ramsay Hunt syndrome) in children: case report and lit- when there is severe disease as evidenced by a large A–a
erature review. Int J Pediatr Otorhinolaryngol. 2012;76:772–
gradient.
776.
Ivermectin would be used to treat Strongyloides
stercoralis hyperinfection, which can cause respiratory
42. ANSWER: C. Mycoplasma genitalium failure in patients who are latently infected and then
his patient has nongonococcal urethritis (NGU) receive corticosteroids, but it is less common in the
based on the clinical diagnosis of urethritis and the United States and is typically manifested with other
negative testing for gonorrhea. he most common symptoms and problems also including diarrhea,
cause of NGU is Chlamydia infection, which causes abdominal pain, gram-negative sepsis, or meningitis
an estimated 15%–40% of all NGU, but this patient related to increased worm burden and transit. hough
also had negative testing for Chlamydia. Other organ- fevers could suggest laring of her giant cell arteritis
isms that cause NGU include Mycoplasma genitalium, such that increased steroids are necessary, her pulmo-
Trichomonas vaginalis, HSV, and occasionally adenovi- nary symptoms and lack of recurrent headaches make
rus. After Chlamydia, M. genitalium is likely the next this less likely. Pulmonary edema is also a consider-
most common cause of NGU at about 15%–25% and ation, but on exam she has no evidence of congestive
is treated with azithromycin. Notably, this organism heart failure and this would not explain the fevers.
is less responsive to doxycycline, and, furthermore, Yale SH, Limper AH. Pneumocystis carinii pneumonia in
azithromycin resistance with this infection is thought patients without acquired immunodeiciency syndrome:
to be increasing in the United States. In patients with associated illness and prior corticosteroid therapy. Mayo
NGU who do not respond to azithromycin, M. geni- Clin Proc. 1996;71:5–13; Kermani TA, Ytterberg SR, War-
talium is the most common cause and retreatment rington KJ. Pneumocystis jiroveci pneumonia in giant cell
with moxiloxacin is indicated. Trichomonas is a less arteritis: a case series. Arthritis Care Res. 2011;63:761–765.
common cause of urethritis but is a consideration in
men who have sex with women and do not respond to 44. ANSWER: C. Patients taking PrEP should have reg-
empiric therapy for NGU with azithromycin or doxy- ular screening for other STIs at least every 6 months,
cycline, particularly in areas where Trichomonas infec- including syphilis, gonorrhea, and chlamydia.
tion is prevalent. Haemophilus ducreyi is the causative he CDC recommends tenofovir/emtricitabine
organism in chancroid, which is an ulcerative STI and once daily for PrEP for HIV prevention for patients
does not typically cause urethritis. with high risk of HIV acquisition, including men
Workowski KA, Bolan GA. Sexually transmitted diseases treat- who have sex with men as well as heterosexual men
ment guidelines, 2015. MMWR Recomm Rep. 2015;64:1–137. and women with increased sexual risk, and injection
drug users. he CDC has extensive online resources
43. ANSWER: B. Trimethoprim-sulfamethoxazole on PrEP for both patients and providers at http://w
his patient has Pneumocystis jirovecii pneumonia, ww.cdc.gov/hiv/risk/prep/. Providers should follow
also still commonly known as PCP. Besides HIV infec- these online guidelines when prescribing PrEP, includ-
tion or AIDS, PCP is most commonly seen in immu- ing baseline and regular HIV testing every 3 months
nocompromised patients treated with corticosteroids. (not 12), regular STI screening, and monitoring renal
CHAPTER 1 Infectious Diseases 25

function because there is risk for nephrotoxicity with common tick-borne illness (Ixodes scapularis), and
tenofovir. Taking tenofovir/emtricitabine for PrEP has no reported travel history to a malaria-endemic
with excellent adherence can reduce the risk of HIV area, so it is presumed that the infection in this case
acquisition by as much as 92% in this patient popula- is babesiosis rather than malaria. Severe babesiosis, as
tion. Since the advent of PrEP, many providers have in this case, is deined as infection resulting in renal,
raised concerns that patients taking PrEP would com- hepatic, or pulmonary end-organ dysfunction, high
pensate with increased sexual risk behaviors, resulting grade parasitemia (>10%), or signiicant hemolysis.
in increased incidence of STIs. However, the PROUD Mild babesiosis may be treated with azithromycin
study of PrEP in a real-world setting demonstrated no and atovaquone (but not with proguanil, so Option
increased incidence of STIs in the PrEP arm, and no A is wrong). Severe babesiosis is generally treated with
evidence of increased risky behaviors in the PrEP arm. quinine (oral quinine or intravenous quinidine when
US Public Health Service. Preexposure prophylaxis for the pre- necessary) and clindamycin. he major risk factors for
vention of HIV infection in the United States—2014. Avail- severe babesiosis in this patient are his history of sple-
able at: http://www.cdc.gov/hiv/pdf/prepguidelines2014. nectomy and rituximab therapy within the last 6–12
pdf. Accessed 29.09.16; McCormack S, Dunn DT, Desai months. His age (i.e., >50 years old) is also a risk fac-
M, et al. Pre-exposure prophylaxis to prevent the acquisition
tor for severe disease. Patients with severe babesiosis
of HIV-1 infection (PROUD): efectiveness results from the
pilot phase of a pragmatic open-label randomised trial. Lan-
should be evaluated for red cell exchange transfusion,
cet. 2016;387:53–60. especially if the patient also has a history of splenec-
tomy and/or recent rituximab therapy. his patient
meets many criteria for red cell exchange transfusion,
45. ANSWER: A. Drug–drug interaction and it should be pursued immediately to prevent fur-
his patient has serotonin syndrome due to interac- ther complications from this disease. Plasmapheresis,
tion of linezolid with duloxetine. Linezolid is a mild steroids, and IVIG (Option D) are not used to treat
monoamine oxidase inhibitor, and when combined babesiosis. Ceftriaxone and doxycycline (Option C)
with certain medications including selective sero- may be used to treat other tick-borne infections (Lyme
tonin reuptake inhibitors, drug-induced serotonin disease and anaplasmosis) but would not be efective
excess can result. he syndrome is characterized by treatment for babesiosis. Notably, babesiosis, Lyme
autonomic instability, cognitive/behavioral changes, disease, and anaplasmosis are transmitted by the same
and neuromuscular excitability; thus the signs and species of tick, I. scapularis, and so coinfection should
symptoms of serotonin syndrome can include tachy- be considered in ill patients. Intravenous artesunate
cardia, hyperthermia, hypertension, agitation, restless- is appropriate therapy for severe malaria but not for
ness, and confusion. his condition can be fatal if not babesiosis.
addressed quickly, which is why there is a black-box Wormser GP, Dattwyler RJ, Shapiro ED, et al. he clinical
warning on linezolid to avoid concurrent administra- assessment, treatment, and prevention of Lyme disease,
tion with selective serotonin reuptake inhibitors, sero- human granulocytic anaplasmosis, and babesiosis: clinical
tonin-norepinephrine reuptake inhibitors, and some practice guidelines by the Infectious Diseases Society of
other serotonin-active medications. Based on the his- America. Clin Infect Dis. 2006;43:1089–1134; Krause PJ,
tory given, there is no evidence of recurrent soft tissue Gewurz BE, Hill D, et al. Persistent and relapsing babe-
infection on physical exam and no reason to suspect siosis in immunocompromised patients. Clin Infect Dis.
2008;46:370–376.
MRSA bacteremia without another source, without
murmurs on cardiac exam, and without evidence of
sepsis (blood pressure is high, rather than low). he 47. ANSWER: A. Blood serologic test for Coccidioides
abdominal examination is normal, and there are no his patient most likely has primary pulmonary
reported abdominal symptoms, so C. diicile infection coccidioidomycosis, or “valley fever.” he features of
is also unlikely. his presentation that are consistent with this diagno-
Lawrence KR, Adra M, Gillman PK. Serotonin toxicity associ- sis are as follows: exposure in an endemic area (hik-
ated with the use of linezolid: a review of postmarketing ing in dry windy climate in the American Southwest),
data. Clin Infect Dis. 2006; 42:1578–1583. somewhat immunosuppressed (due to his psoriasis
and methotrexate treatment), and acute-onset illness
46. ANSWER: B. Quinine and clindamycin and con- with fever, fatigue, chest pain, and cough, which all
sideration of red cell exchange transfusion persisted despite antibiotic therapy. he chest x-ray
his patient has severe babesiosis. Intraerythro- may show an iniltrate, or it may be normal. Labo-
cytic parasites (protozoa) are diagnostic of either ratory indings often also show new eosinophilia, but
malaria (caused by multiple diferent Plasmodium spe- this may also be normal. his patient has ongoing
cies) or babesiosis (in the United States most often symptoms, but they are not worsening, so his disease is
caused by Babesia microti). his patient has trav- relatively mild and uncomplicated, despite his under-
eled to Nantucket, where babesiosis is a relatively lying treatment with methotrexate. Serologic testing
26 C HA P T E R 1 Infectious Diseases

for Coccidioides (IgM and IgG) is often diagnostic in K production and therefore hypocoagulable state,
these cases—if the serology is positive, then the patient rather than hypercoagulable state.
most likely has active Coccidioides infection, because Lee CA, hrasher KA. Diiculties in anticoagulation manage-
serologic reactivity tends to wane within months of ment during coadministration of warfarin and rifampin.
an infection. However, it sometimes takes months for Pharmacotherapy. 2001;21:1240–1246.
antibody tests to be positive, so sputum culture may
be the only way to diagnose early infection. Many 49. ANSWER: D. Francisella tularensis
patients with this infection will have self-limited dis- Tularemia is the infection caused by Francisella
ease, often even subclinical. Directed treatment, with tularensis, which is a gram-negative bacterium. People
antifungals such as itraconazole or liposomal ampho- may develop tularemia infection after contact with
tericin, is generally only required in severe, dissemi- infected animals or via insect vectors such as ticks.
nated, or prolonged infection. Most of the cases in the United States are reported
Although the fungal antigen 1,3-β-d-glucan (Option in Arkansas, Missouri, Kansas, South Dakota, Okla-
B) can be positive in active coccidioidomycosis, it is homa, and California. After exposure, patients may
not a speciic test because it can also be positive in the be asymptomatic, or they may develop any one of the
setting of other invasive fungal infections such as dis- six major clinical forms of tularemia: ulceroglandular
seminated candidiasis and histoplasmosis. CT imag- tularemia, glandular tularemia, oculoglandular tulare-
ing would similarly be abnormal but not speciic for mia, pharyngeal (oropharyngeal) tularemia, typhoidal
coccidioidomycosis. Hilar lymph node biopsy (Option tularemia, or pneumonic tularemia. Ulceroglandular
D) may also reveal the diagnosis but would not be an and pneumonic tularemia are the two most commonly
initial diagnostic test of choice. Serum galactomannan diagnosed forms of the disease. his patient has pneu-
(Option E) is only helpful for making the diagnosis monic tularemia. Primary pneumonic disease occurs
of aspergillosis typically in an immunocompromised after inhalation of the organism from a source, such
host. as an infected animal—this is most common among
Galgiani JN, Ampel NM, Blair JE, et al. 2016 Infectious Dis- farmers, sheep workers, landscapers, and hunters.
eases Society of America (IDSA) clinical practice guideline Pneumonic tularemia may present similarly to pneu-
for the treatment of coccidioidomycosis. Clin Infect Dis. monic plague (caused by Yersinia pestis), except that
2016;63:e112–e146. pulmonary disease often consists of peribronchial inil-
trates or lobar consolidations in pneumonic tularemia,
48. ANSWER: E. Drug–drug interaction of warfarin whereas it is often rounded and cavitated pulmonary
with rifampin iniltrates or nodules in setting of pneumonic plague.
Rifampin is not a commonly prescribed antibi- Patients may have some laboratory abnormalities,
otic, but it is used in several speciic situations and including elevated or depressed WBC and abnormal
has numerous drug–drug interactions because it is liver function tests, but laboratory indings may also
a potent inducer of cytochrome P450 3A. Patient be normal. Patients may develop respiratory failure
medication lists should be reviewed carefully for and empyema with pneumonic tularemia, and infec-
potential interactions whenever rifampin is started, tion may even spread beyond the lungs, with compli-
and a plan for monitoring or adjustment in treat- cations such as meningitis and endocarditis. Diagnosis
ment should be made whenever possible. In this case, is primarily by serologies and clinical suspicion, but
rifampin induces the metabolism of warfarin so that the organism may be cultured (with diiculty) if cul-
the patient has approximately half the previously tures are speciically requested on cysteine-containing
available warfarin dose after rifampin is started. If supportive media. When tularemia is suspected, the
the INR is not monitored carefully in order to facili- laboratory must be notiied in order to take special
tate warfarin dose adjustments, then patients will precautions because it is highly transmissible in the
quickly become subtherapeutic in this setting. Even laboratory setting. Treatment is with doxycycline or
with proper monitoring and dose adjustments, some ciproloxacin for mild disease, and with streptomycin
patients are unable to achieve therapeutic warfarin or gentamicin for severe disease.
levels while on rifampin and may require concurrent Tropheryma whipplei is the etiologic agent in Whip-
treatment with enoxaparin or other medications. ple disease, which commonly presents with abdominal
his patient had a recent transesophageal echocar- pain, diarrhea, weight loss, and joint pains. Babesia
diogram, which did not show any valvular vegeta- microti is a parasite that is transmitted by ticks primar-
tions or perivalvular abscess, and the blood cultures ily in New England and to some extent in the upper
on this presentation are negative, so there is no evi- Midwest; babesiosis is characterized by fever, anemia,
dence to suggest treatment failure at this point. Gen- thrombocytopenia, and often respiratory distress, but
tamicin has many associated potential toxicities, but this patient had no anemia or parasites visualized on
it does not cause stroke. Loss of gut lora while on initial laboratory indings. Borrelia lonestari is thought
antibiotic therapy often leads to decreased vitamin to be the causative agent in Southern tick–associated
CHAPTER 1 Infectious Diseases 27

rash illness (STARI), which is a Lyme disease–like which is more often polymicrobial and purulent as
infection described in patients in the southeastern and compared with this presentation of “streptococcal
south-central United States. Anaplasma phagocytophi- gangrene.” Necrotizing soft tissue infections can also
lum causes anaplasmosis, formerly known as human be caused by polymicrobial lora, which can include
granulocytic ehrlichiosis, which is another tick-borne E. coli, other gram-negative pathogens, and anaerobes;
illness that often presents with fever, headache, and lab this type of necrotizing skin and soft tissue infection
abnormalities but rarely causes pulmonary disease. most commonly occurs in the groin and genital area
Centers for Disease Control and Prevention. Tularemia— (“Fournier gangrene”). Clostridium species including
United States, 2001-2010. MMWR Morb Mortal Wkly Rep. C. perfringens and C. septicum also cause necrotiz-
2013;62:963–966. ing skin and soft tissue infection with gas formation
(which was not present in this nonclostridial case) and
50. ANSWER: C. Group A streptococci myonecrosis. Aeromonas hydrophila is a rare cause of
his patient has a necrotizing soft tissue infec- necrotizing skin and soft tissue infection associated
tion with myonecrosis after blunt trauma. his is with trauma in or exposed to fresh water. he history
most often caused by group A beta-hemolytic strep- for this patient was not consistent with this pathogen.
tococci. Treatment is emergent surgical debridement Stevens DL, Bisno AL, Chambers HF, et al. Practice guidelines
and aggressive antibiotics, preferably with high-dose for the diagnosis and management of skin and soft tissue
IV penicillin and clindamycin to reduce toxin forma- infections: 2014 update by the Infectious Diseases Society
tion once the pathogen is identiied. If antibiotics are of America. Clin Infect Dis. 2014;59:147–159.
started in the absence of Gram stain results, then a
more broad-spectrum regimen would be appropriate Acknowledgment
until culture data are available, including vancomycin
and piperacillin-tazobactam or a carbapenem. Staphy- he authors and editors gratefully acknowledge the con-
lococci, including methicillin-resistant or methicillin- tributions of the previous authors—Michael Calderwood,
sensitive, can cause necrotizing soft tissue infections, Rebeca Plank, Dylan Tierney, and Sigal Yawetz.
2
Hematology and Oncology
AMY BESSNOW AND ANN S. LACASCE

1. An 80-year-old man presents with several months of symptoms. Her CA 19-9 is drawn and found to be
epigastric pain, weight loss, fatigue, and dyspnea. he normal. You refer the patient for magnetic resonance
patient was diagnosed with pernicious anemia 5 years cholangiopancreatography (MRCP), which conirms a
ago and has been receiving regular vitamin B12 injec- 1.9-cm mucinous lesion involving the main pancreatic
tions. At that time, he had a colonoscopy, which was duct, with classic features of an intraductal papillary
notable for extensive sigmoid diverticulosis. His com- mucinous neoplasm of the pancreas (IPMN).
plete blood count (CBC) today reveals a hematocrit of What is the proper next step in management?
24% and a mean cell volume (MCV) of 75 fL. A. Follow-up CT scan in 6 months
What malignancy is most likely responsible for his B. Follow-up MRCP in 6 months
symptoms? C. Referral to a pancreatic surgeon for resection
A. Myelodysplastic syndrome D. See the patient in 6–12 months to assess symp-
B. Gastric cancer toms, but obtain no further imaging studies.
C. Colon cancer E. his is a benign inding; no speciic follow-up is
D. Mucosa-associated lymphoid tissue (MALT) lym- required.
phoma
E. Pancreatic cancer 4. A 35-year-old woman originally from the Dominican
Republic comes to the clinic for her irst visit. She has
2. A 45-year-old man presents with back pain, jaundice, no signiicant medical problems and has had two chil-
and weight loss of 20 pounds. A CT scan of the abdo- dren. After the birth of her second child 3 years ago, she
men reveals a large mass in the pancreatic head. His was told to take iron tablets twice daily. Aside from an
cancer antigen (CA) 19-9 is elevated at 496 U/mL. he oral contraceptive, this is her only medication. Results
patient’s family history is notable for the following. His of her laboratory studies are shown in Table 2.1.
father was diagnosed with colon cancer at age 72. His What is the most appropriate management?
mother is an only child and was diagnosed with breast A. Phlebotomy for hemochromatosis
cancer at age 51. His maternal grandfather died of pan- B. Continue current iron therapy and initiate workup
creatic cancer at age 55. His sister was diagnosed with for chronic inlammatory process.
ovarian cancer at 52 and is currently in remission. C. Switch therapy from oral iron sulfate to iron dex-
he patient is most likely to have which of the fol- tran.
lowing hereditary cancer syndromes? D. Discontinue iron therapy; send for ferritin and
A. Lynch syndrome hemoglobin electrophoresis.
B. BRCA2 mutation
C. CDKN2A mutation 5. A 74-year-old man with diabetes mellitus controlled
D. Li-Fraumeni syndrome with an oral agent presents for routine follow-up.
E. CDH1 mutation His other medical problems include hypertension,
for which he takes an angiotensin-converting enzyme
3. A 60-year-old healthy woman undergoes a CT scan (ACE) inhibitor, and benign prostatic hypertrophy.
of the abdomen for signs and symptoms of nephroli- On review of his records, you note that his hema-
thiasis. he scan conirms a small right kidney stone tocrit has been gradually declining over the past 3
but also reveals a 2-cm cystic lesion involving the main years. Results of his laboratory studies are shown in
pancreatic duct. After spontaneously passing the kid- Table 2.2.
ney stone, the patient follows up in your clinic stat- What is the most likely etiology of his anemia?
ing that she feels entirely well. She denies jaundice, A. Combined iron and vitamin B12 deiciency
abdominal or back pain, weight loss, or any other B. Medication efect from the ACE inhibitor

28
CHAPTER 2 Hematology and Oncology 29

TABLE TABLE
2.1 Laboratory Results for Question 4 2.3 Complete Blood Count Results for Question 6

White blood cell 4600/mm3 (4000–10,000) Hemoglobin 5.6 g/dL


count
Hematocrit 17%
Hematocrit 35% (36–48)
Mean cell volume 123 fL
Mean cell volume 66 fL (80–95)
White blood cell count 3500/mm3
Red blood cell 6.0 (4.2–5.6)
count Platelets 70,000/µL

Platelets 256,000/mm3 (150,000–450,000) Bilirubin 2.3 mg/dL

Iron 150 µg/dL (40–159)


Total iron-binding 275 µg/dL (250–400) TABLE
capacity 2.4 Laboratory Results for Question 7

Hemoglobin 11.6 g/dL


TABLE Hematocrit 31%
2.2 Laboratory Results for Question 5
Mean cell volume 83 fL
White blood cell 7000/mm3 (4000–10,000) Reticulocytes 7.0%
count
Platelets 220,000/mm3
Hematocrit 28% (36–48)
White blood cell count 6500/mm3
Mean cell volume 84 fL (80–95)
Blood smear Spherocytes, increased
Red blood cell dis- 15 (10–14.5) reticulocytes
tribution width
Coombs test Negative
Platelets 340,000/mm3 (150,000–450,000)
Blood urea nitrogen 35 mg/dL (9–25)
Creatinine 1.9 mg/dL (0.7–1.3) years. She thinks that she has always tired more easily than
her friends, and she has been told several times that she is
Lactate 230 U/L (107–231)
dehydrogenase anemic. She has been treated on several occasions with
iron pills, but not in the past 2 years. Her physical exami-
nation reveals that she has scleral icterus and a spleen tip
palpable below the left costal margin (Table 2.4).
C. Erythropoietin deiciency he patient is most likely to respond to which of
D. Anemia due to marrow replacement by metastatic the following?
prostate cancer A. Corticosteroids
B. Intravenous iron
6. A 50-year-old man presents complaining of dyspnea on C. Splenectomy
exertion. He was in a car accident 5 years ago with mul- D. Eculizumab
tiple abdominal injuries, resulting in a splenectomy and
a resection of several feet of his terminal ileum. After a 8. A 39-year-old man with sickle cell anemia is admitted
prolonged recovery, he returned to work and his normal for management of pneumonia. He presented with a
activities. For the last 4–6 months, he has had trouble 2-day history of a dry, nonproductive cough and fever
climbing the stairs to his bedroom without stopping to to 101°F, and he was found to have a right lower lobe
catch his breath. He is on no medications, has not lost iniltrate. On admission, his oxygen saturation was
weight, and has a well-balanced diet (see Table 2.3 for 94% on room air, and he was not short of breath.
CBC). Peripheral smear showed macrocytic erythrocytes, He is placed on cefuroxime and given intravenous
hypersegmented neutrophils, and decreased platelets. hydration (Table 2.5). One day later, he complains of
What is the most likely diagnosis? increasing shortness of breath and is found to have an
A. Myelodysplastic syndrome oxygen saturation of 86% on room air. Chest radiog-
B. Vitamin B12 deiciency raphy reveals bilateral lower lobe opacities.
C. Sideroblastic anemia What are the best next steps?
D. Hemolytic anemia A. Continue current antibiotic coverage and admin-
ister supplemental O2.
7. An 18-year-old woman is referred for the evaluation of B. Continue current antibiotic coverage, administer
mild jaundice. She thinks she has had it intermittently for supplemental O2, and obtain a V/Q scan.
30 C HA P T E R 2 Hematology and Oncology

TABLE 10. In which of the following patients should yearly


2.5 Laboratory Results for Question 8 screening with low-dose computed tomography for
lung cancer not be considered?
CBC on Admission A. A 47-year-old woman with a 2-pack-per-day active
White blood 18,000/mm3 (4000–10,000) tobacco use habit and a history of lung cancer in
cell count both her mother and father
Hematocrit 21% (36–48) B. A 57-year-old woman with a history of a 1-pack-
per-day tobacco use habit from ages 15 to 50
Platelets 247,000/mm3 (150,000–
450,000) C. A 70-year-old man with a history of asbestos expo-
sure and 50-pack-year history of cigarette smoking
who quit smoking 12 years ago
D. A 75-year-old woman who has smoked a half-pack
TABLE Laboratory Studies in the Emergency Room of cigarettes per day since age 30
2.6 for Question 9 E. A 60-year-old man with a 50-pack-year history of
cigarette smoking whose last cigarette was after a
White blood cell 14,000/mm3 (4000–10,000)
count heart attack at age 50
Hematocrit 20% (36–48)
11. A 47-year-old woman required emergency craniotomy
Platelets 317,000/mm3 (150,000–450,000) and repair of an aneurysm. On postoperative day 2, she
Reticulocyte count 0.3% (0.6–2.8) develops left calf pain and is found to have thrombus
in the popliteal vessel. Intravenous (IV) unfractionated
Total bilirubin 1.8 mg/dL (0.2–1.2)
heparin is started given her recent neurosurgery. Two
Direct bilirubin 0.3 mg/dL (0.0–0.3) days later, warfarin is started. On postoperative day 7,
Lactate dehydro- 240 U/L (135–225) she is found to have right leg swelling and dyspnea.
genase Evaluation reveals pulmonary emboli and thrombus in
the right common femoral vein. Her platelet count is
noted to be 87,000/µL, down from 320,000/µL at the
time of surgery.
C. Continue current antibiotic coverage, administer Next steps in her management include:
supplemental O2, and transfuse packed red blood A. Immediately stop all heparin exposure, including
cells (PRBCs). IV line lush.
D. Add coverage for atypical organisms and adminis- B. Start IV direct thrombin inhibitor such as argatro-
ter supplemental O2. ban or bivalirudin.
E. Add coverage for atypical organisms, administer C. Give 10 mg IV vitamin K to reverse warfarin.
supplemental O2, and exchange transfuse. D. Test for heparin-platelet factor 4 antibodies.
E. All of the above
9. A 54-year-old African American man with sickle cell
anemia presents to the emergency room with a 2-day 12. A 32-year-old woman 8 weeks postpartum is seen
history of fatigue and pain in his back and lower by her obstetrician for fevers, fatigue, and bruis-
extremities. He scores his pain at 5 out of 10. He has ing. In addition to elevated temperature as well as
had episodes of pain in the past and is well known some bruises and petechiae on examination, she is
to the emergency department staf. His temperature is found to have a hematocrit of 21%, platelet count
99°F, heart rate is 108 beats per minute, blood pressure of 23,000/µL, and creatinine of 2.4 mg/dL. Values
is 120/70 mm Hg, and oxygen saturation on room air at time of delivery were normal. While waiting to get
is 96% (see Table 2.6). He is started on intravenous more laboratory serum tests, she develops right arm
(IV) normal saline and morphine. weakness and confusion. She is sent directly to the
What is the best next step? emergency room and admitted to the intensive care
A. Admit to the hospital and start broad-spectrum unit.
antibiotics. What should your immediate next step be?
B. Admit to the hospital and transfuse packed red A. Request neurology consult for electroencephalo-
blood cells (PRBCs). gram (EEG).
C. Admit to the hospital and arrange for emergent B. Transfuse with platelets.
RBC exchange transfusion. C. Give IV luids for dehydration.
D. Manage pain aggressively and discharge home D. Check peripheral smear and ind schistocytes; ini-
with close hematology follow-up. tiate plasmapheresis.
E. Manage pain aggressively and discharge home on E. Send for ADAMTS13 level and wait for results
oral antibiotics. before treating.
CHAPTER 2 Hematology and Oncology 31

13. A 39-year-old man is seen by a new primary care physi- C. Mixing study
cian. “von Willebrand disease (vWD)” is listed in his past D. Platelet aggregation studies
medical history, but the patient does not know many E. d-Dimer
details of this. He had an episode of gastrointestinal (GI)
bleeding in college requiring hospitalization and RBC 17. A 63-year-old man with type 2 diabetes mellitus and
transfusion when taking nonsteroidal antiinlammatory hypertension presents with recurrent painless hema-
drugs (NSAIDs). He was told he has vWD and to avoid turia. He undergoes a CT scan of the abdomen and
aspirin and NSAIDs. Review of systems is negative for pelvis, which shows an enhancing 6.3-cm left kid-
recent episodes of bleeding, bruising, or epistaxis. His ney mass highly suspicious for a renal cell carcinoma.
father has had some bleeding episodes in the past. here were multiple retroperitoneal enlarged lymph
What tests do you send to evaluate for vWD? nodes, and the lung bases showed four or ive pul-
A. Factor VIII activity monary nodules, with the largest measuring 1.1 cm.
B. vWF antigen level he patient has a good performance status. His CBC
C. Ristocetin cofactor and comprehensive metabolic panel (CMP) results
D. vWF multimer gel electrophoretic analysis were within normal limits, except for a hemoglobin of
E. All of the above 11.5 g/dL.
he next steps would include all except:
14. A 43-year-old woman undergoes elective cholecystectomy A. Biopsy of the kidney mass
for gallstones. Her past medical history is unremarkable, B. Biopsy of the lung lesions
and she is on no medications. On postoperative day 4, C. Chest CT, brain magnetic resonance imaging
she develops left lower-extremity calf pain and is found to (MRI), bone scan
have popliteal vein thrombus by compression ultrasound D. Initiating sunitinib or pazopanib
(US). She has no past history of thrombosis. She weighs E. Proceed with cytoreductive nephrectomy.
68 kg and has normal renal function.
Which is the best treatment option? 18. A 72-year-old man who is an active smoker presents
A. Compression stockings with dysuria and vague abdominal/pelvic pain. His
B. Aspirin 325 mg once daily urinalysis reveals 10–20 RBCs/high-power ield. He
C. Low-molecular-weight heparin overlapping with undergoes an abdominal ultrasound that is unremark-
warfarin targeting international normalized ratio able except for potential thickening of the bladder
1.5–2.0 wall on the left side. Cystoscopy shows a 5-cm bladder
D. Rivaroxaban 15 mg twice daily × 3 weeks, then 20 mass. he patient undergoes transurethral resection of
mg once daily bladder tumor (TURBT), which reveals high-grade
E. Intravenous unfractionated heparin (IV UFH) urothelial cancer with focal invasion into the lamina
propria. No muscle is available in the specimen.
15. he patient in Question 14 has no past history of he next step would be:
thrombosis, despite two pregnancies. Her father had a A. Radical cystectomy
deep vein thrombosis (DVT) at age 43, and his father B. Partial cystectomy (left bladder wall)
had a pulmonary embolism in his 60s. She has two C. Proceed with bacille Calmette-Guérin (BCG)
daughters, ages 12 and 15. immunotherapy.
She should be tested for thrombophilia: D. Repeat TURBT
A. True E. Chemoradiation to the bladder mass
B. False
19. A 51-year-old woman presents with recurrent urinary
16. A 32-year-old woman undergoes laparoscopic resection tract infections (UTIs) for the past 6 months and pel-
of a benign complex ovarian cyst by her gynecologist. vic pain for 6 weeks. A CT scan of her pelvis shows a
One week later she presents complaining of increasing large bladder mass. Cystoscopy followed by TURBT
lower back and abdominal pain. On examination, she shows a poorly diferentiated urothelial carcinoma.
has a 7-cm × 4-cm subfascial hematoma from the umbi- Tumor is invading into the muscularis propria. Chest
licus to the right lower quadrant and a 6-cm × 3-cm and abdominal CT do not show distant metastases or
suprapubic ecchymosis. Her prothrombin time (PT) enlarged lymph nodes. Her serum creatinine is 0.75
is 12.3 seconds (normal 11.2–13.4 seconds), and her mg/dL. Her bone scan is normal. he patient has an
activated partial thromboplastin time (aPTT) is 72.4 excellent performance status.
seconds (normal 23.8–36.6 seconds). Repeat testing Which is the correct statement?
reveals PT 12.8 seconds and aPTT 81.5 seconds. A. Neoadjuvant cisplatin-based chemotherapy is
What is the next test to evaluate an isolated elevated indicated based on a 15% absolute reduction in
aPTT? death.
A. Factor XIII level B. Adjuvant cisplatin-based chemotherapy is indi-
B. Fibrinogen cated based on a 15% absolute reduction in death.
32 C HA P T E R 2 Hematology and Oncology

C. Neoadjuvant cisplatin-based chemotherapy is Which of the following is not a risk associated with
indicated based on a 5% absolute reduction in androgen deprivation therapy?
death. A. Osteoporosis and bone fracture
D. Adjuvant cisplatin-based chemotherapy is indi- B. Prolactinoma
cated based on a 5% absolute reduction in death. C. Hot lashes
E. Proceed with radical cystectomy. D. Anemia
E. Increase in subcutaneous adipose tissue
20. A 59-year-old man, previously healthy and taking only
one 81-mg aspirin per day, was referred for prostate 23. Immune therapy with programmed death receptor 1
biopsy after a screening showed a serum prostate-spe- (PD-1) inhibition can be efective and is often well tol-
ciic antigen (PSA) of 4.8 ng/mL and unremarkable erated in patients with a variety of advanced malignan-
digital rectal examination (DRE). A 12-core needle cies. In which of the following patients should immune
biopsy was performed, revealing two cores on the left therapy with PD-1 inhibition not be considered?
with Gleason 3 + 3 adenocarcinoma involving up to A. A patient with metastatic squamous cell carci-
20% of each core. All other cores showed no evidence noma of the epiglottis that recurs after platinum-
of disease. he patient asks you about imaging to com- based therapy and unknown tumor programmed
plete his diagnostic workup. death receptor ligand 1 (PD-L1) expression
What would you advise? B. A patient with newly diagnosed metastatic non-
A. Endorectal coil MRI only small cell lung cancer, poor performance status,
B. CT scan of the abdomen/pelvis only and 10% tumor PD-L1 expression
C. Bone scan only C. A patient with metastatic nonsmall cell lung can-
D. CT scan of the abdomen/pelvis and bone scan cer refractory to platinum-based chemotherapy
E. No further workup necessary and 0% tumor PD-L1 expression
D. A 30-year-old patient with recurrent classic Hodg-
21. A 61-year-old man presents with a history of hyper- kin lymphoma after autologous transplant and
tension and hyperlipidemia, and he is taking aspirin, brentuximab vedotin
hydrochlorothiazide, and simvastatin daily. He has been E. An elderly patient with metastatic melanoma and
undergoing PSA screening. His digital rectal examina- heart disease
tion has been normal, but a recent PSA of 5.4 ng/mL
prompted a prostate biopsy. Biopsy demonstrated 1 24. A 76-year-old woman with stage IV nonsmall cell lung
of 12 cores positive for Gleason 4 + 3 prostate cancer cancer has been receiving nivolumab for the past 3
involving 15% of the core. His urologist ordered an months after her disease recurred following chemother-
endorectal coil MRI that showed a 1.6-cm left-sided apy with carboplatin and pemetrexed. She was feeling
lesion suspicious for tumor. here was no evidence quite well, but a few days after her last infusion she calls
of extracapsular extension, seminal vesicle involve- to report rapidly progressive shortness of breath and dry
ment, or enlarged lymph nodes. A bone scan was also cough over the past 24 hours. On evaluation, she has
ordered, but it showed no evidence of metastases. he no fever, she is tachycardic to the 110s, her blood pres-
patient is anxious about the side efects associated with sure is mildly elevated at 155/90 mm Hg, and she her
prostate cancer treatment and asks for your guidance. respiratory rate is 22 breaths per minute. Her lungs are
Which of the following is not a recommended clear to auscultation bilaterally, and she is able to speak
option for primary management of this patient’s pros- in full sentences easily, although she becomes dyspneic
tate cancer? walking briskly across the room. She is referred to the
A. Active surveillance ER, where her chest radiograph is unremarkable and
B. Radical prostatectomy electrocardiogram shows only tachycardia. Pulmonary
C. External beam radiation therapy embolism (PE)-protocol CT is without visualized clot
D. Interstitial brachytherapy but is notable for difuse ground-glass opacities and
E. All of the options above are acceptable. areas of mildly increased interstitial markings.
Which of the following interventions is indicated?
22. A 66-year-old man presented with hip pain and was A. A course of antibiotics for suspected pneumonia
ultimately diagnosed with prostate cancer that had B. Enoxaparin for suspected small-vessel PE
metastasized to the ribs and pelvic bones. His PSA at C. Prednisone at least 1 mg/kg or equivalent
the time of diagnosis was 180 ng/mL. He was started D. A and C
on androgen deprivation therapy with leuprolide, a E. A course of antibiotics and inhaled corticosteroids
gonadotropin-releasing hormone (GnRH) agonist.
After 6 months on treatment, his pain resolved and his 25. A 40-year-old woman with a strong family history
PSA has decreased to 0.8 ng/mL. He presents to the of breast cancer is considering taking tamoxifen for
clinic to discuss side efects of his therapy. chemoprevention. She calls to discuss potential side
CHAPTER 2 Hematology and Oncology 33

efects of the medication. She reports she does not did not demonstrate other nodules or mediastinal or
want to experience premature menopause, weight hilar adenopathy.
gain, or depression. What is the most appropriate next step for follow-
What do you advise her? up of the lung nodule?
A. Tamoxifen causes none of these side efects. A. Repeat chest CT in 3 months
B. She should take raloxifene instead. B. Repeat chest CT in 6 months
C. She should take exemestane instead. C. Repeat chest CT in 2 years
D. Tamoxifen may cause weight gain and depression, D. PET/CT now
but not premature menopause. E. Referral for CT-guided biopsy
E. She should wait until after menopause to start
tamoxifen. 29. A 58-year-old woman with minimal prior medical
history and with good performance status is found to
26. A 68-year-old man with a 40-pack-year smoking have a new 2.2-cm peripheral right lower lobe mass.
history and chronic obstructive pulmonary disease Complete staging evaluation demonstrates only the
(COPD) presented to his local emergency room after solitary lung mass, with no other sites of disease in the
falling on his bathroom rug and hitting his head and mediastinum or distantly. CT-guided biopsy conirms
right ribs. A noncontrast head CT scan did not show a diagnosis of nonsmall cell lung cancer, adenocarci-
evidence of bleeding. A chest radiograph did not show noma histology.
any broken ribs, but it did incidentally show a spicu- What is the most appropriate next step in manage-
lated right upper lobe lung mass measuring 3.6 × 2.4 ment?
cm. A subsequent biopsy conirms nonsmall cell lung A. Chemotherapy
cancer. B. Cryoablation
Which of the following radiographic studies is not C. Repeat CT scan in 6 months
recommended as part of a subsequent staging evalua- D. Stereotactic radiation
tion? E. Surgical resection
A. Brain MRI with gadolinium
B. Chest CT with IV contrast 30. A 25-year-old woman presents with a dry cough, inter-
C. Positron emission tomography (PET)-CT mittent drenching night sweats, and difuse pruritus
D. Skeletal survey with plain ilms without identiiable rash. On physical examination,
she appears tired. Her vital signs are notable for a tem-
27. A 63-year-old woman who had smoked two packs of perature of 99.9°F, heart rate of 110 beats per minute,
cigarettes daily for 30 years presents with several weeks blood pressure of 100/80 mm Hg, respiratory rate of
of increasing cough and dyspnea with exertion, fol- 18 breaths per minute, and oxygen saturation of 98%
lowed over the last few days with lethargy and swelling on room air. Her physical examination is notable for
of her face and arms. Initial evaluation in the emer- the absence of peripheral lymphadenopathy. Her chest
gency room reveals a serum sodium of 120 mEq/L, is clear to auscultation bilaterally. Cardiac examination
blood urea nitrogen (BUN) 17 mg/dL, and creatinine reveals tachycardia with a soft systolic ejection mur-
0.8 mg/dL. Chest CT with IV contrast is negative for mur. She had no splenomegaly. Her lower extremities
pulmonary embolus but demonstrates a 5.5-cm right are notable for linear excoriations. Laboratory stud-
hilar mass with associated mediastinal adenopathy and ies reveal a white blood cell count of 14,300/µL with
compression of the superior vena cava; furthermore, 80% neutrophils and 5% lymphocytes, 10% eosino-
there appear to be multiple osseous and hepatic metas- phils, and 5% monocytes. Her basic metabolic panel
tases. is normal.
What is the most likely diagnosis? What is the next most appropriate step in her man-
A. Breast cancer agement?
B. Bronchial carcinoid tumor A. Obtain a lactate dehydrogenase (LDH) level.
C. Non-Hodgkin lymphoma B. Order a PET/CT scan.
D. Small cell lung cancer C. Refer to dermatology for evaluation.
E. hymoma D. Obtain a chest x-ray.
E. Send for Epstein-Barr virus (EBV) serologies.
28. A 70-year-old man with a history of kidney stones
and a 20-pack-year smoking history underwent CT of 31. A 70-year-old man presents to the emergency depart-
the abdomen as part of an evaluation for lank pain. ment for evaluation of upper respiratory symptoms
CT demonstrated recurrence of nephrolithiasis but with low-grade fevers. He appears clinically well. His
also demonstrated an incidental, peripheral, 5-mm physical examination is notable for a 1-cm palpable
left lower lobe (LLL) lung nodule. Chest CT was per- lymph nodes in the bilateral neck and axilla. His
formed and again showed the small LLL nodule but spleen tip is palpable just below the left costal margin.
34 C HA P T E R 2 Hematology and Oncology

Laboratory studies reveal a white blood cell count of thyroid-stimulating hormone (TSH) of 2.2 mIU/L,
18,000/µL with 75% lymphocytes, with smudge cells, and prolactin of 360 ng/mL. You suspect a prolac-
15% neutrophils, and 5% monocytes. His chemistries tinoma and order a pituitary MRI, which reveals a
are normal, including LDH. 10-mm × 12-mm pituitary lesion. here is no cavern-
What is the next most appropriate step? ous sinus invasion or compression of the optic nerves
A. Obtain peripheral blood low cytometry. by the pituitary tumor.
B. Refer for excisional lymph node biopsy. You advise the patient that the initial treatment of
C. Order a PET/CT to evaluate the best node for choice is:
biopsy. A. Medical therapy with a somatostatin analogue
D. Perform a bone marrow aspiration and biopsy. B. Observation only
E. Treat with antibiotics and recheck CBC in 1 C. Transsphenoidal pituitary tumor resection
month. D. Medical therapy with a dopamine agonist
E. Radiation therapy to the pituitary
32. A 65-year-old woman presents complaining of a mass
in the left neck without fevers, night sweats, weight 35. Which of the following antidepressants is preferred in
loss, or localizing symptoms. On examination, she has women taking tamoxifen for hormone receptor–positive
normal vital signs and is found to have a 2.5-cm left breast cancer?
anterior cervical node, a 2-cm left axillary node, and A. Paroxetine
a 3-cm right inguinal node. Her examination is oth- B. Fluoxetine
erwise normal. Laboratory studies, including a CBC C. Venlafaxine
with diferential, basic metabolic panel, and LDH, are D. Bupropion
normal. E. Duloxetine
What is the next most appropriate step in her man-
agement? 36. Which of the following patients would be a good can-
A. Refer to interventional radiology for a needle didate for novel oral anticoagulant (NOAC) therapy?
biopsy of the most easily accessible lymph node. A. A 65-year-old woman with rheumatic heart dis-
B. Refer to otolaryngology for ine-needle aspiration ease and a metal prosthetic mitral valve
of the neck mass. B. A 35-year-old woman with a newly diagnosed
C. Refer to a surgeon for an excisional lymph node PE, history of multiple second-trimester preg-
biopsy. nancy losses, and positive antiphospholipid anti-
D. Obtain a PET/CT scan. bodies
E. Send for peripheral blood low cytometry. C. A 60-year-old man with new-onset nonvalvular
atrial ibrillation and a history of a diverticular
33. A 66-year-old postmenopausal woman with hyper- bleed 5 years ago
tension and hypothyroid is diagnosed with stage D. A 60-year-old man with a newly diagnosed PE and
IA ER/PR-positive, HER2/neu-negative invasive metastatic nonsmall cell lung cancer
ductal carcinoma. She undergoes lumpectomy fol- E. A 60-year-old man with newly diagnosed DVT
lowed by radiation, and chemotherapy is deferred and renal insuiciency
based on a low risk of recurrence and low likelihood
of beneit. She is started on an aromatase inhibitor 37. A 64-year-old woman presents with abdominal bloat-
(AI). ing, early satiety, and constipation. On bimanual
For which of the following is she now at signii- exam, an enlarged right ovary is noted. Ultrasound
cantly increased risk due to her aromatase inhibitor? demonstrates a complex 10-cm × 8-cm mass highly
A. DVT/PE concerning for malignancy as well as a moderate
B. Hypothyroid amount of free luid in the pelvis.
C. Osteoporosis Which of the following is not potentially indicated?
D. Endometrial cancer A. Abdominal/pelvic CT
E. Glucose intolerance B. Laboratory testing for cancer antigen 125 (CA-125)
C. Colonoscopy
34. A 35-year-old woman with a history of regular men- D. Referral to gynecologic oncology for consideration
ses presents with complaints of bilateral galactorrhea. of surgery
She notes that her menses have been more irregular E. Referral to interventional radiology for needle-
over the last year and that her last menstrual period guided biopsy of the adnexal mass
was approximately 6 weeks ago. She reports occa-
sional headaches but otherwise feels well. She does 38. A 22-year-old man presents with headache and
not take any medication or herbal supplements. peripheral vision loss. Brain imaging reveals a large sel-
Laboratory testing reveals a negative pregnancy test, lar mass with both solid and cystic components that is
CHAPTER 2 Hematology and Oncology 35

compressing the optic chiasm. Humphrey visual ield presents to Student Health for evaluation. Her physi-
testing demonstrates bitemporal hemianopsia. He cal examination is notable for scattered, quarter-sized
has no previous medical history, his body mass index bruises on her upper arms and thighs. Laboratory stud-
(BMI) is 20 kg/m2, and he is taking no medications. ies are notable for a white blood cell count of 500/µL,
His initial pituitary functional evaluation revealed hemoglobin of 7 g/dL, and platelets of 50,000/µL.
central hypogonadism and central hypothyroidism. A peripheral smear is notable for 80% large atypical
His adrenal function was normal. MRI characteris- cells with folded, bilobed, kidney-shaped nucleoli and
tics suggested the mass was a craniopharyngioma. He elongated, bluish-red rods within the cytoplasm.
was taken to surgery to remove the pituitary mass in What translocation is the deining feature of this
attempts to decompress the optic chiasm and restore disease?
his vision. He did well in the immediate postopera- A. t(11;14)
tive period, but within 24 hours, he began to develop B. t(15;17)
polyuria. His urine output increased to 400 mL/h and C. t(9;22)
was very dilute with a urine speciic gravity of less than D. t(8;14)
1.001. He complained of extreme thirst. His serum E. t(14;18)
sodium increased to 148 mEq/L, and his fasting glu-
cose was elevated at 106 mg/dL. 41. A 65-year-old man with past medical history of hyper-
What diagnosis are you suspecting in this patient? tension presents to his primary care physician (PCP)
A. Syndrome of inappropriate antidiuretic hormone with several weeks of fatigue and dyspnea on exer-
secretion (SIADH) tion (DOE). He notes feeling more winded during
B. Diabetes mellitus, type 2 his 3-mile walks. His vital signs are stable. His blood
C. Central diabetes insipidus pressure is 120/60 mm Hg, and his O2 saturation is
D. Nephrogenic diabetes insipidus 97% on room air. He had no hepatosplenomegaly. His
E. None of the above complete blood count is notable for white blood cell
count of 4000/µL with 60% neutrophils, hemoglobin
39. A 55-year-old woman was involved in a motor vehi- of 9 g/dL, and platelets of 110,000/µL. he laboratory
cle accident as an unrestrained driver. Although she indings are notable for a vitamin B12 of 700 pg/mol,
did not lose consciousness, she sustained a signiicant folate of 20 ng/mL, TSH of 1.2 mIU/mL, testosterone
head trauma and was brought by ambulance to the of 400 ng/dL, and erythropoietin of 90 U/L. A bone
emergency department. She was evaluated and found marrow biopsy is done and is remarkable for dysplas-
to have a normal neurologic examination, but due to tic erythroid precursors and megakaryocytes with 1%
the mechanism of her accident, brain imaging by non- blasts. Cytogenetics are normal. He has 2 points—1
contrast CT scan was performed. She was incidentally for Hgb of 9 g/dL and 1 for normal cytogenetics—
found to have a large sellar lesion, estimated at approx- based on his Revised International Prognostic Scoring
imately 2 cm in greatest diameter. No acute intracra- System (IPSS-R) score.
nial hemorrhage was identiied. She had a laceration What would be the most appropriate recommenda-
on her forehead that was repaired, and because she was tion for therapy for his myelodysplastic syndrome?
otherwise clinically stable, she was discharged with a A. Allogeneic stem cell transplant
plan to follow up with her primary care provider for B. Induction chemotherapy with an anthracycline +
further evaluation of this pituitary mass. cytarabine
In evaluating a patient with a newly discovered C. Lenalidomide
pituitary mass, what are the important initial clinical D. Erythropoiesis-stimulating agent (ESA)
considerations? E. No treatment necessary
A. Evaluation for mass efects (headaches, visual loss,
cranial nerve abnormalities) 42. A 35-year-old woman presents to her PCP for her
B. Evaluation for pituitary hormonal hypersecretion annual visit. She has been feeling well. She denies
C. Evaluation for pituitary hormonal hypofunction fevers, night sweats, and weight loss. She denies GI
D. All of the above symptoms, including nausea, vomiting, abdominal
E. None of the above pain, and early satiety. She denies easy bleeding, bruis-
ing, or history of clots. On examination, her spleen
40. A 22-year-old college student notes increasing fatigue is palpable two ingerbreadths below the costal mar-
over the past few weeks that she attributes to staying gin. A complete blood count is notable for a white
up late studying for inal examinations. Over the past blood cell count of 60,000/µL, hemoglobin of 10 g/
2–3 days she has noted several new bruises on her dL, and platelets of 600,000/µL. Review of the periph-
upper and lower extremities. She does not recall ante- eral smear is notable for a large percentage of basophils
cedent trauma leading to the bruising. his morning and eosinophils, along with myelocytes and metamy-
she developed a bloody nose lasting 20 minutes. She elocytes. JAK2 V617F mutation is negative.
36 C HA P T E R 2 Hematology and Oncology

What is the most appropriate next step in manage- E. Despite normal vWF levels, pregnant women with
ment? vWD have increased risk of bleeding during deliv-
A. Start hydroxyurea. ery and should be treated prophylactically with
B. Check blood cultures and start empiric antibiotics Humate-P.
for possible infection.
C. Start high-dose aspirin (325 mg twice daily). 45. A 68-year-old lifelong-nonsmoking woman in excel-
D. Send peripheral blood for cytogenetics to evaluate lent general health is brought to the ER after seizure-like
for the t(9;22) chromosome translocation. activity was observed by her family during dinner. She is
E. Determine a leukocyte alkaline phosphatase (LAP) somnolent on arrival but within 30 minutes is alert and
score. oriented and without focal neurologic indings. Noncon-
trast head CT shows no bleed, and brain MRI with and
43. A 60-year-old woman with past medical history of without contrast demonstrates three foci of enhancement
hypertension and COPD is brought to the emergency measuring up to 2 cm and located in the right parietal,
room by her daughter. She is lethargic and is having occipital, and left frontal lobes. here is surrounding vaso-
diiculty walking. She has noted increasing dii- genic edema, particularly of the largest lesion in the left
culty breathing over the past few hours. Her physi- frontal cortex. However, there is no mass efect or shift.
cal examination is notable for temperature of 100.7°F, CT of the chest, abdomen, and pelvis demonstrate a spic-
blood pressure of 150/70 mm Hg, respiratory rate of ulated left upper lobe lung mass and multiple bony lesions.
24 breaths per minute, and O2 saturation of 88% on Biopsy of the left upper lobe mass demonstrates poorly dif-
room air. She has diiculty with inger-to-nose and ferentiated adenocarcinoma consistent with lung primary.
rapid alternating movements. She has ine crackles Which step would not be included in the manage-
bilaterally. A complete blood count is notable for a ment of this patient?
white blood cell count of 110 × 106, hemoglobin of 8, A. Radiation oncology consultation for consideration
and platelets of 75,000/µL. A chest x-ray shows bilat- of urgent whole-brain radiotherapy (WBRT)
eral iniltrates. B. Intravenous dexamethasone
What would be the appropriate next step in man- C. Initiation of levetiracetam
agement of this patient? D. Radiation oncology consultation for consideration
A. Give a unit of PRBCs. of stereotactic radiosurgery (SRS)
B. Start antibiotics for possible pneumonia and fol- E. Molecular testing of tumor specimen for targe-
low for symptom improvement before initiating table mutations
more invasive procedures.
C. Start steroids and antibiotics for COPD lare. 46. In which of the following patients would you be most
D. Start leukapheresis while establishing diagnosis. concerned about initiating therapy with ibrutinib for
E. Establish diagnosis and start appropriate therapy. symptomatic chronic lymphocytic leukemia (CLL)?
A. A patient with bulky nodal disease and anemia
44. A 32-year-old woman presents in active labor on a Fri- B. A patient with a p53 mutation and rapid disease
day evening. She tells the admitting obstetrician who recurrence after ludarabine, cyclophosphamide,
is covering for a group practice that she has a history of and rituximab
heavy periods and mucosal bleeding and was told she C. A patient with a history of pneumocystis pneumo-
has von Willebrand disease. She uses a nasal inhaler nia and recurrent varicella zoster virus infections
at the start of her periods, and it reduces bleeding. D. A patient with poorly controlled atrial ibrillation
Her mother and sister have similar bleeding histories. and platelets of 20,000/µL
You ind laboratory values done prior to her preg- E. A patient with a history of immune colitis
nancy showing vWF antigen of 42%, vWF activity of
38%, and factor VIII of 49%. All of these are below 47. Your patient is a 56-year-old schoolteacher who pre-
the lower limits of normal. here is a note stating that sented with about 2 months’ history of intermittent
the values are compatible with type I von Willebrand blood in the toilet bowl with bowel movements. A
disease. colonoscopy is performed that demonstrates a mass in
Which of the following statements is/are true? the midsigmoid, and biopsy conirms adenocarcinoma.
A. von Willebrand factor levels go down during preg- He undergoes laparoscopic hemicolectomy, and inal
nancy. pathology reveals a 3-cm, moderately diferentiated
B. Placement of an epidural catheter is contraindi- adenocarcinoma invading through the muscle layer into
cated in a patient with von Willebrand disease. the serosa with two of nine lymph nodes positive.
C. he correct treatment would be infusion of recom- What would be the next step?
binant coagulation factor VIIa (NovoSeven). A. Reoperate for more complete lymph node dissection.
D. von Willebrand factor levels become normal dur- B. Referral to medical oncologist for consideration of
ing pregnancy in patients with type I disease. chemotherapy
CHAPTER 2 Hematology and Oncology 37

C. Follow-up colonoscopy in 1 year lumbosacral back pain for 2 weeks. His physical exam-
D. Referral to radiation oncologist for postoperative ination is normal except for localized tenderness over
radiation L4, with no neurologic deicits.
What is the best next step?
48. A 42-year-old man newly diagnosed with Burkitt lym- A. Increase his scheduled pain medications and see
phoma presents to the emergency room 3 days after him for follow-up in 1 month.
beginning chemotherapy. He complains of decreased B. Arrange for an urgent MRI of the spine to evaluate
urine output. Laboratory data show serum potassium for possible cord compression.
of 6.5 mg/dL, BUN of 48 mg/dL, serum creatinine of C. Arrange for radiation therapy within 24 hours for
2.4 mg/dL, uric acid of 10.9 mg/dL, and phosphorus probable cord compression.
of 6.4 mg/dL, with uric acid crystals on his urinalysis. D. Emergency neurosurgical evaluation for suspected
He is already on allopurinol. spinal cord compression
What treatment is least likely to be efective at this
point? 50. Which one of the following statements is true regard-
A. Hydration and furosemide, monitoring urine out- ing superior vena cava (SVC) syndrome?
put and weight to maintain euvolemia A. Establishing a tissue diagnosis is the most impor-
B. Acute management of hyperkalemia with calcium tant step in management of most patients with
gluconate and insulin and dextrose SVC syndrome.
C. Initiation of hemodialysis if diuresis does not occur B. SVC syndrome should be treated with IV throm-
D. A recombinant uricase, such as rasburicase bolytics and stenting of the vena cava.
E. Increase in dose of allopurinol C. Diagnosis is usually established by chest x-ray,
then venography if needed.
49. A 69-year-old man with prostate cancer and wide- D. Breast cancer and germ cell tumors are the most
spread bone metastases complains of worsening common malignant causes of SVC syndrome.

Chapter 2 Answers
1. ANSWER: B. Gastric cancer and pancreatic cancer, is highly suggestive of BRCA
Pernicious anemia is an autoimmune disorder that mutation. Both BRCA1 and BRCA2 mutations are
is characterized by autoantibodies against intrinsic associated with increased risk of pancreatic cancer,
factor, destruction of gastric parietal cells, vitamin but BRCA2 is much more strongly associated. BRCA2
B12 deiciency, and gastric achlorhydria. Over time, mutation is thought to be responsible for approxi-
the condition progresses to chronic atrophic gastritis, mately 15% of familial pancreatic cancer. Although
intestinal metaplasia, dysplasia, and ultimately gastric Lynch syndrome (hereditary nonpolyposis colorec-
adenocarcinoma. he patient described here has devel- tal cancer [HNPCC]) is associated with a modestly
oped occult upper gastrointestinal (GI) bleeding from increased risk of pancreatic cancer, his father’s diagno-
gastric cancer, stemming from long-standing perni- sis of colon cancer at age 72 is not highly suggestive of
cious anemia and atrophic gastritis. Myelodysplastic Lynch syndrome. Li-Fraumeni syndrome results from
syndrome has not been associated with pernicious ane- a mutation in the p53 tumor suppressor gene, and it
mia, nor is it associated with iron deiciency. Although is most notably associated with early diagnosis of sar-
the patient likely has iron-deiciency anemia, perni- coma, brain tumors, breast cancer, and leukemia, but
cious anemia is also not associated with colon can- not pancreatic cancer. Mutations in CDKN2A result
cer. Additionally, an advanced colon cancer would in the familial atypical multiple-mole melanoma syn-
be unlikely, given the patient’s colonoscopy indings drome, which is associated with markedly increased
5 years previously. MALT lymphoma is highly linked risk of melanoma and pancreatic cancer. Germline
to Helicobacter pylori infection, not pernicious anemia. mutations in CDH1 (E-cadherin) are associated with
Although some reports indicate an increased risk of a very high risk of difuse gastric cancer and breast can-
pancreatic cancer associated with pernicious anemia, cer, but not pancreatic cancer.
pancreatic cancer would not cause iron-deiciency
anemia. 3. ANSWER: C. Referral to a pancreatic surgeon for
resection
2. ANSWER: B. BRCA2 mutation he patient was incidentally found on an imaging
his patient presents with a likely diagnosis of pan- study to have a main duct IPMN. IPMNs are prema-
creatic adenocarcinoma. In addition to his young age, lignant lesions of the pancreatic ductal epithelium,
his family history raises concern for a possible heredi- which have the potential to progress to pancreatic
tary cancer syndrome. he patient’s maternal family adenocarcinoma. IPMNs involving the side branches
history, including early breast cancer, ovarian cancer, of the pancreatic ductal system (so-called branch duct
Another random document with
no related content on Scribd:
distributing a Project Gutenberg™ electronic work under this
agreement, disclaim all liability to you for damages, costs and
expenses, including legal fees. YOU AGREE THAT YOU HAVE NO
REMEDIES FOR NEGLIGENCE, STRICT LIABILITY, BREACH OF
WARRANTY OR BREACH OF CONTRACT EXCEPT THOSE
PROVIDED IN PARAGRAPH 1.F.3. YOU AGREE THAT THE
FOUNDATION, THE TRADEMARK OWNER, AND ANY
DISTRIBUTOR UNDER THIS AGREEMENT WILL NOT BE LIABLE
TO YOU FOR ACTUAL, DIRECT, INDIRECT, CONSEQUENTIAL,
PUNITIVE OR INCIDENTAL DAMAGES EVEN IF YOU GIVE
NOTICE OF THE POSSIBILITY OF SUCH DAMAGE.

1.F.3. LIMITED RIGHT OF REPLACEMENT OR REFUND - If you


discover a defect in this electronic work within 90 days of receiving it,
you can receive a refund of the money (if any) you paid for it by
sending a written explanation to the person you received the work
from. If you received the work on a physical medium, you must
return the medium with your written explanation. The person or entity
that provided you with the defective work may elect to provide a
replacement copy in lieu of a refund. If you received the work
electronically, the person or entity providing it to you may choose to
give you a second opportunity to receive the work electronically in
lieu of a refund. If the second copy is also defective, you may
demand a refund in writing without further opportunities to fix the
problem.

1.F.4. Except for the limited right of replacement or refund set forth in
paragraph 1.F.3, this work is provided to you ‘AS-IS’, WITH NO
OTHER WARRANTIES OF ANY KIND, EXPRESS OR IMPLIED,
INCLUDING BUT NOT LIMITED TO WARRANTIES OF
MERCHANTABILITY OR FITNESS FOR ANY PURPOSE.

1.F.5. Some states do not allow disclaimers of certain implied


warranties or the exclusion or limitation of certain types of damages.
If any disclaimer or limitation set forth in this agreement violates the
law of the state applicable to this agreement, the agreement shall be
interpreted to make the maximum disclaimer or limitation permitted
by the applicable state law. The invalidity or unenforceability of any
provision of this agreement shall not void the remaining provisions.

1.F.6. INDEMNITY - You agree to indemnify and hold the


Foundation, the trademark owner, any agent or employee of the
Foundation, anyone providing copies of Project Gutenberg™
electronic works in accordance with this agreement, and any
volunteers associated with the production, promotion and distribution
of Project Gutenberg™ electronic works, harmless from all liability,
costs and expenses, including legal fees, that arise directly or
indirectly from any of the following which you do or cause to occur:
(a) distribution of this or any Project Gutenberg™ work, (b)
alteration, modification, or additions or deletions to any Project
Gutenberg™ work, and (c) any Defect you cause.

Section 2. Information about the Mission of


Project Gutenberg™
Project Gutenberg™ is synonymous with the free distribution of
electronic works in formats readable by the widest variety of
computers including obsolete, old, middle-aged and new computers.
It exists because of the efforts of hundreds of volunteers and
donations from people in all walks of life.

Volunteers and financial support to provide volunteers with the


assistance they need are critical to reaching Project Gutenberg™’s
goals and ensuring that the Project Gutenberg™ collection will
remain freely available for generations to come. In 2001, the Project
Gutenberg Literary Archive Foundation was created to provide a
secure and permanent future for Project Gutenberg™ and future
generations. To learn more about the Project Gutenberg Literary
Archive Foundation and how your efforts and donations can help,
see Sections 3 and 4 and the Foundation information page at
www.gutenberg.org.
Section 3. Information about the Project
Gutenberg Literary Archive Foundation
The Project Gutenberg Literary Archive Foundation is a non-profit
501(c)(3) educational corporation organized under the laws of the
state of Mississippi and granted tax exempt status by the Internal
Revenue Service. The Foundation’s EIN or federal tax identification
number is 64-6221541. Contributions to the Project Gutenberg
Literary Archive Foundation are tax deductible to the full extent
permitted by U.S. federal laws and your state’s laws.

The Foundation’s business office is located at 809 North 1500 West,


Salt Lake City, UT 84116, (801) 596-1887. Email contact links and up
to date contact information can be found at the Foundation’s website
and official page at www.gutenberg.org/contact

Section 4. Information about Donations to


the Project Gutenberg Literary Archive
Foundation
Project Gutenberg™ depends upon and cannot survive without
widespread public support and donations to carry out its mission of
increasing the number of public domain and licensed works that can
be freely distributed in machine-readable form accessible by the
widest array of equipment including outdated equipment. Many small
donations ($1 to $5,000) are particularly important to maintaining tax
exempt status with the IRS.

The Foundation is committed to complying with the laws regulating


charities and charitable donations in all 50 states of the United
States. Compliance requirements are not uniform and it takes a
considerable effort, much paperwork and many fees to meet and
keep up with these requirements. We do not solicit donations in
locations where we have not received written confirmation of
compliance. To SEND DONATIONS or determine the status of
compliance for any particular state visit www.gutenberg.org/donate.

While we cannot and do not solicit contributions from states where


we have not met the solicitation requirements, we know of no
prohibition against accepting unsolicited donations from donors in
such states who approach us with offers to donate.

International donations are gratefully accepted, but we cannot make


any statements concerning tax treatment of donations received from
outside the United States. U.S. laws alone swamp our small staff.

Please check the Project Gutenberg web pages for current donation
methods and addresses. Donations are accepted in a number of
other ways including checks, online payments and credit card
donations. To donate, please visit: www.gutenberg.org/donate.

Section 5. General Information About Project


Gutenberg™ electronic works
Professor Michael S. Hart was the originator of the Project
Gutenberg™ concept of a library of electronic works that could be
freely shared with anyone. For forty years, he produced and
distributed Project Gutenberg™ eBooks with only a loose network of
volunteer support.

Project Gutenberg™ eBooks are often created from several printed


editions, all of which are confirmed as not protected by copyright in
the U.S. unless a copyright notice is included. Thus, we do not
necessarily keep eBooks in compliance with any particular paper
edition.

Most people start at our website which has the main PG search
facility: www.gutenberg.org.

This website includes information about Project Gutenberg™,


including how to make donations to the Project Gutenberg Literary
Archive Foundation, how to help produce our new eBooks, and how
to subscribe to our email newsletter to hear about new eBooks.

You might also like